Set 1 of 3 - Midterm

Pataasin ang iyong marka sa homework at exams ngayon gamit ang Quizwiz!

A 52-year-old female patient is going through menopause and asks the nurse about estrogen replacement for its cardioprotective benefits. What is the best response by the nurse?

"Current evidence indicates that estrogen is ineffective as a cardioprotectant; estrogen is actually potentially harmful and is no longer a recommended therapy."

The nurse reviews discharge instructions with a client who underwent a left groin cardiac catheterization 8 hours ago. Which instructions should the nurse include?

"Do not bend at the waist, strain, or lift heavy objects for the next 24 hours."

A community health nurse teaches a group of seniors about modifiable risk factors that contribute to the development of peripheral arterial disease (PAD). The nurse knows that the teaching was effective based on which of the following statements?

"I will need to stop smoking because the nicotine causes less blood to flow to my hands and feet."

The nurse is assessing vital signs on a client who is 3 months status post myocardial infarction (MI). While the healthcare provider is examining the client, the client's spouse approaches the nurse and states "We are too afraid he will have another heart attack, so we just don't have sex anymore." What is the nurse's best response?

"The physiologic demands are greatest during orgasm and are equivalent to walking 3 to 4 miles per hour on a treadmill."

The nurse is assisting a patient with peripheral arterial disease to ambulate in the hallway. What should the nurse include in the education of the patient during ambulation?

"Walk to the point of pain, rest until the pain subsides, then resume ambulation."

Which of the following findings is not a significant risk factor for heart disease?

(HDL), 80 mg/dL

Heath Catheterization Post-Procedure Care

1. Assess VS & O2 sat q15 minutes for 2-3 hours 2. Assess cardiac status / monitor pattern 3. Assess groin puncture site q15 minutes for 2-3 hours - Look for bleeding including behind site 4. Assess pedal pulses q15 minutes for 2-3 hours 5. Immobilize leg with puncture site = bed rest 6. Encourage fluids - flush dye out of body via kidneys - More fluids if renal insufficiency

Which blood chemistry values (minerals) are essential to monitor for hear function?

1. Calcium (8.6 to 10.2 mg/dL) 2. Magnesium (1.3 to 2.3 mEq/dL) ** 3. Potassium (3.5 to 5 mEq/L) **

What are the 2 major sources of cholesterol?

1. Dietary Intake 2. Synthesis in Liver

What are some cardiac test for determining cardiac rhythms?

1. EKG/ECG 2. Haltar Monitor - Wear for 24 hours 3. Event Cardiac Monitoring - Loop recorder placed invasively 4. Trans-Telephonic Monitoring - Place phone over receiver in chest

What are the 2 types of cardiac stress testing?

1. Exercise Stress Test 2. Pharmacologic Stress Test

Heath Catheterization Pre-Procedure Care

1. Informed Consent 2. NPO 8-12 hours before procedure 3. Allergies (Dyes & Medications) 4. Routine Meds 5. Baseline Assessment 6. Locate & mark peripheral pulses 7. Pre-procedure medicines

What are some nursing diagnoses for heart catheterization?

1. Risk for bleeding/hemorrhage at puncture site 2. Risk for alteration in perfusion 3. Risk for cardiac dysrhythmias 4. Risk for decreased renal function

What are some factors that lower HDL?

1. Smoking 2. Obesity 3. Diabetes 4. Physical Inactivity

When administering heparin anticoagulant therapy, the nurse needs to make certain that the activated partial thromboplastin time (aPTT) is within the therapeutic range of:

1.5 to 2.5 times the baseline control

Hypertension is repeated blood pressure measurements exceeding:

140/90 mm Hg.

A healthy level of serum cholesterol would be a reading of:

160 to 190 mg/dL

What is the normal CVP?

2-6 mm Hg

A goal of dilation in PTCA is to increase blood flow through the artery's lumen and achieve a residual stenosis of less than

20%

In the United States, about 1 million people will have an acute myocardial infarction each year. Of these 1 million, what percentage will die?

25%

What is normal PA pressure?

25/10 mm Hg

Patient education includes telling someone who takes nitroglycerin sublingually that he or she should take 1, then go quickly to the nearest emergency department if no relief has been obtained after taking ______ tablet(s) at 5-minute intervals.

3

The nurse advises a patient that sublingual nitroglycerin should alleviate angina pain within:

3 to 4 minutes

What is the normal flow rate of the NS infusion?

3-5 mL/min

What pressure is the NS infusion bag pressurized to?

300 mm Hg (Usually 500 cc NS infusion bag)

21. The nurse is caring for an acutely ill patient who has central venous pressure monitoring in place. What intervention should be included in the care plan of a patient with CVP in place? A) Apply antibiotic ointment to the insertion site twice daily. B) Change the site dressing whenever it becomes visibly soiled. C) Perform passive range-of-motion exercises to prevent venous stasis. D) Aspirate blood from the device once daily to test pH.

499 *: B Feedback: Gauze dressings should be changed every 2 days or transparent dressings at least every 7 days and whenever dressings become damp, loosened, or visibly soiled. Passive ROM exercise is not indicated and it is unnecessary and inappropriate to aspirate blood to test it for pH. Antibiotic ointments are contraindicated.

28. When hemodynamic monitoring is ordered for a patient, a catheter is inserted into the appropriate blood

502 vessel or heart chamber. When assessing a patient who has such a device in place, the nurse should check which of the following components? Select all that apply. A) A transducer B) A flush system C) A leveler D) A pressure bag E) An oscillator *: A, B, D Feedback: To perform hemodynamic monitoring, a CVP, pulmonary artery, or arterial catheter is introduced into the appropriate blood vessel or heart chamber. It is connected to a pressure monitoring system that has several components. Included among these are a transducer, a flush system, and a pressure bag. A pressure monitoring system does not have a leveler or an oscillator.

30. The nurse is doing discharge teaching with a patient who has coronary artery disease. The patient asks why he has to take an aspirin every day if he doesnt have any pain. What would be the nurses best response?

503 A) Taking an aspirin every day is an easy way to help restore the normal function of your heart. B) An aspirin a day can help prevent some of the blockages that can cause chest pain or heart attacks. C) Taking an aspirin every day is a simple way to make your blood penetrate your heart more freely. D) An aspirin a day eventually helps your blood carry more oxygen that it would otherwise. *: B Feedback: An aspirin a day is a common nonprescription medication that improves outcomes in patients with CAD due to its antiplatelet action. It does not affect oxygen carrying capacity or perfusion. Aspirin does not restore cardiac function.

32. The patient has a homocysteine level ordered. What aspects of this test should inform the nurses care? Select all that apply. A) A 12-hour fast is necessary before drawing the blood sample. B) Recent inactivity can depress homocysteine levels. C) Genetic factors can elevate homocysteine levels.

504 D) A diet low in folic acid elevates homocysteine levels. E) An ECG should be performed immediately before drawing a sample. *: A, C, D Feedback: Genetic factors and a diet low in folic acid, vitamin B6 , and vitamin B12 are associated with elevated homocysteine levels. A 12-hour fast is necessary before drawing a blood sample for an accurate serum measurement. An ECG is unnecessary and recent inactivity does not influence the results of the test.

39. The student nurse is preparing a teaching plan for a patient being discharged status post MI. What should the student include in the teaching plan? (Mark all that apply.) A) Need for careful monitoring for cardiac symptoms B) Need for carefully regulated exercise

507 C) Need for dietary modifications D) Need for early resumption of prediagnosis activity E) Need for increased fluid intake *: A, B, C Feedback: Dietary modifications, exercise, weight loss, and careful monitoring are important strategies for managing three major cardiovascular risk factors: hyperlipidemia, hypertension, and diabetes. There is no need to increase fluid intake and activity should be slowly and deliberately increased.

The incidence of coronary artery disease tends to be equal for men and women after the age of

55 years

What is the normal MAP?

70-90 mm Hg

What is the normal pulmonary artery wedge pressure?

8-12 mm Hg

33. A patient with a complex cardiac history is scheduled for transthoracic echocardiography. What should the nurse teach the patient in anticipation of this diagnostic procedure? A) The test is noninvasive, and nothing will be inserted into the patients body. B) The patients pain will be managed aggressively during the procedure. C) The test will provide a detailed profile of the hearts electrical activity. D) The patient will remain on bed rest for 1 to 2 hours after the test.

: A Feedback: Before transthoracic echocardiography, the nurse informs the patient about the test, explaining that it is painless. The test does not evaluate electrophysiology and bed rest is unnecessary after the procedure.

18. The nurse is caring for a patient admitted with angina who is scheduled for cardiac catheterization. The patient is anxious and asks the reason for this test. What is the best response? A) Cardiac catheterization is usually done to assess how blocked or open a patients coronary arteries are. B) Cardiac catheterization is most commonly done to detect how efficiently a patients heart muscle contracts. C) Cardiac catheterization is usually done to evaluate cardiovascular response to stress. D) Cardiac catheterization is most commonly done to evaluate cardiac electrical activity.

: A Feedback: Cardiac catheterization is usually used to assess coronary artery patency to determine if revascularization procedures are necessary. A thallium stress test shows myocardial ischemia after stress. An ECG shows the electrical activity of the heart.

29. The critical care nurse is caring for a patient who has been experiencing bradycardia after cardiovascular surgery. The nurse knows that the heart rate is determined by myocardial cells with the fastest inherent firing rate. Under normal circumstances where are these cells located? A) SA node B) AV node C) Bundle of His D) Purkinje cells

: A Feedback: The heart rate is determined by the myocardial cells with the fastest inherent firing rate. Under normal circumstances, the SA node has the highest inherent rate (60 to 100 impulses per minute).

37. A patients declining cardiac status has been attributed to decreased cardiac action potential. Interventions will be aimed at restoring what aspect of cardiac physiology? A) The cycle of depolarization and repolarization B) The time it takes from the firing of the SA node to the contraction of the ventricles C) The time between the contraction of the atria and the contraction of the ventricles D) The cycle of the firing of the AV node and the contraction of the myocardium

: A Feedback: This exchange of ions creates a positively charged intracellular space and a negatively charged extracellular space that characterizes the period known as depolarization. Once depolarization is complete, the exchange of ions reverts to its resting state; this period is known as repolarization. The repeated cycle of depolarization and repolarization is called the cardiac action potential.

34. A critical care nurse is caring for a patient with a hemodynamic monitoring system in place. For what complications should the nurse assess? Select all that apply. A) Pneumothorax B) Infection C) Atelectasis D) Bronchospasm E) Air embolism

: A, B, E * 505 Feedback: Complications from use of hemodynamic monitoring systems are uncommon, but can include pneumothorax, infection, and air embolism. Complications of hemodynamic monitoring systems do not include atelectasis or bronchospasm.

25. The nurses assessment of an older adult client reveals the following data: Lying BP 144/82 mm Hg; sitting BP 121/69 mm Hg; standing BP 98/56 mm Hg. The nurse should consequently identify what nursing diagnosis in the patients plan of care? A) Risk for ineffective breathing pattern related to hypotension B) Risk for falls related to orthostatic hypotension C) Risk for ineffective role performance related to hypotension D) Risk for imbalanced fluid balance related to hemodynamic variability

: B * 501 Feedback: Orthostatic hypotension creates a significant risk for falls due to the dizziness and lightheadedness that accompanies it. It does not normally affect breathing or fluid balance. The patients ability to perform normal roles may be affected, but the risk for falls is the most significant threat to safety.

38. A patient has been scheduled for cardiovascular computed tomography (CT) with contrast. To prepare the patient for this test, what action should the nurse perform? A) Keep the patient NPO for at least 6 hours prior to the test. B) Establish peripheral IV access. C) Limit the patients activity for 2 hours before the test. D) Teach the patient to perform incentive spirometry.

: B Feedback: An IV is necessary if contrast is to be used to enhance the images of the CT. The patient does not need to fast or limit his or her activity. Incentive spirometry is not relevant to this diagnostic test.

31. The physician has ordered a high-sensitivity C-reactive protein (hs-CRP) drawn on a patient. The results of this test will allow the nurse to evaluate the role of what process that is implicated in the development of atherosclerosis? A) Immunosuppression B) Inflammation C) Infection D) Hemostasis

: B Feedback: High-sensitivity CRP is a protein produced by the liver in response to systemic inflammation. Inflammation is thought to play a role in the development and progression of atherosclerosis.

2. During a shift assessment, the nurse is identifying the clients point of maximum impulse (PMI). Where will the nurse best palpate the PMI? A) Left midclavicular line of the chest at the level of the nipple B) Left midclavicular line of the chest at the fifth intercostal space C) Midline between the xiphoid process and the left nipple D) Two to three centimeters to the left of the sternum

: B Feedback: The left ventricle is responsible for the apical beat or the point of maximum impulse, which is normally palpated in the left midclavicular line of the chest wall at the fifth intercostal space.

26. A brain (B-type) natriuretic peptide (BNP) sample has been drawn from an older adult patient who has been experienced vital fatigue and shortness of breath. This test will allow the care team to investigate the possibility of what diagnosis? A) Pleurisy B) Heart failure C) Valve dysfunction D) Cardiomyopathy

: B Feedback: The level of BNP in the blood increases as the ventricular walls expand from increased pressure, making it a helpful diagnostic, monitoring, and prognostic tool in the setting of HF. It is not specific to cardiomyopathy, pleurisy, or valve dysfunction.

8. The physical therapist notifies the nurse that a patient with coronary artery disease (CAD) experiences a much greater-than-average increase in heart rate during physical therapy. The nurse recognizes that an increase in heart rate in a patient with CAD may result in what? A) Development of an atrial-septal defect B) Myocardial ischemia C) Formation of a pulmonary embolism D) Release of potassium ions from cardiac cells

: B Feedback: Unlike other arteries, the coronary arteries are perfused during diastole. An increase in heart rate shortens diastole and can decrease myocardial perfusion. Patients, particularly those with CAD, can develop myocardial ischemia. An increase in heart rate will not usually result in a pulmonary embolism or create electrolyte imbalances. Atrial-septal defects are congenital.

24. The nurse is relating the deficits in a patients synchronization of the atrial and ventricular events to his diagnosis. What are the physiologic characteristics of the nodal and Purkinje cells that provide this synchronization? Select all that apply. A) Loop connectivity B) Excitability C) Automaticity D) Conductivity E) Independence

: B, C, D Feedback: Three physiologic characteristics of two types of specialized electrical cells, the nodal cells and the Purkinje cells, provide this synchronization: automaticity, or the ability to initiate an electrical impulse; excitability, or the ability to respond to an electrical impulse; and conductivity, the ability to transmit an electrical impulse from one cell to another. Loop connectivity is a distracter for this question. Independence of the cells has nothing to do with the synchronization described in the scenario.

27. A lipid profile has been ordered for a patient who has been experiencing cardiac symptoms. When should a lipid profile be drawn in order to maximize the accuracy of results? A) As close to the end of the day as possible B) After a meal high in fat C) After a 12-hour fast D) Thirty minutes after a normal meal

: C Feedback: Although cholesterol levels remain relatively constant over 24 hours, the blood specimen for the lipid profile should be obtained after a 12-hour fast.

15. A patient has had a myocardial infarction and has been diagnosed as having damage to the layer of the heart responsible for the pumping action. You are aware that the damage occurred where? A) Endocardium B) Pericardium C) Myocardium D) Visceral pericardium

: C Feedback: The myocardium is the layer of the heart responsible for the pumping action.

35. The nurse is caring for a patient who has central venous pressure (CVP) monitoring in place. The nurses most recent assessment reveals that CVP is 7 mm Hg. What is the nurses most appropriate action? A) Arrange for continuous cardiac monitoring and reposition the patient. B) Remove the CVP catheter and apply an occlusive dressing. C) Assess the patient for fluid overload and inform the physician. D) Raise the head of the patients bed and have the patient perform deep breathing exercise, if possible.

: C Feedback: The normal CVP is 2 to 6 mm Hg. Many problems can cause an elevated CVP, but the most common is due to hypervolemia. Assessing the patient and informing the physician are the most prudent actions. Repositioning the patient is ineffective and removing the device is inappropriate.

14. The cardiac care nurse is reviewing the conduction system of the heart. The nurse is aware that electrical conduction of the heart usually originates in the SA node and then proceeds in what sequence? A) SA node to bundle of His to AV node to Purkinje fibers B) SA node to AV node to Purkinje fibers to bundle of His C) SA node to bundle of His to Purkinje fibers to AV node D) SA node to AV node to bundle of His to Purkinje fibers

: D * 496 Feedback: The normal electrophysiological conduction route is SA node to AV node to bundle of HIS to Purkinje fibers.

36. A critical care nurse is caring for a patient with a pulmonary artery catheter in place. What does this catheter measure that is particularly important in critically ill patients? A) Pulmonary artery systolic pressure B) Right ventricular afterload C) Pulmonary artery pressure D) Left ventricular preload

: D Feedback: Monitoring of the pulmonary artery diastolic and pulmonary artery wedge pressures is particularly important in critically ill patients because it is used to evaluate left ventricular filling pressures (i.e., left ventricular preload). This device does not directly measure the other listed aspects of cardiac function. * 506

40. The nurse is caring for a patient who is undergoing an exercise stress test. Prior to reaching the target heart rate, the patient develops chest pain. What is the nurses most appropriate response? A) Administer sublingual nitroglycerin to allow the patient to finish the test. B) Initiate cardiopulmonary resuscitation. C) Administer analgesia and slow the test. D) Stop the test and monitor the patient closely.

: D Feedback: Signs of myocardial ischemia would necessitate stopping the test. CPR would only be necessary if signs of cardiac or respiratory arrest were evident.

20. A critically ill patient is admitted to the ICU. The physician decides to use intra-arterial pressure monitoring. After this intervention is performed, what assessment should the nurse prioritize in the plan of care? A) Fluctuations in core body temperature B) Signs and symptoms of esophageal varices C) Signs and symptoms of compartment syndrome D) Perfusion distal to the insertion site

: D Feedback: The radial artery is the usual site selected. However, placement of a catheter into the radial artery can further impede perfusion to an area that has poor circulation. As a result, the tissue distal to the cannulated artery can become ischemic or necrotic. Vigilant assessment is thus necessary. Alterations in temperature and the development of esophageal varices or compartment syndrome are not high risks.

The nurse is assessing a patient who reports feeling "light-headed." When obtaining orthostatic vital signs, what does the nurse determine is a significant finding?

A heart rate of more than 20 bpm above the resting rate

The nurse needs to be alert to assess for clinical symptoms of possible postoperative complications of PTCA, which include:

All - abrupt closure of the artery., arterial dissection, coronary artery vasospasm

Lumen narrowing with atherosclerosis is caused by

All 3: atheroma formation on the intima, scarred endothelium, and thrombus formation

A candidate for coronary artery bypass grafting (CABG) must meet which of the following criteria?

All these - A blockage that cannot be treated by PTCA b. Greater than 60% blockage in the left main coronary artery. c. Unstable angina.

Which assessment test must a patient pass before inserting an arterial line? Why?

Allen Test (Ensure patency of both radial and ulnar artery)

Which of the following medications is considered a thrombolytic?

Alteplase

The need for surgical intervention in coronary artery disease (CAD) is determined by the:

Amount of stenosis in the coronary arteries, myocardial area served by the stenotic artery, and occurrence of previous infarction related to the affected artery

A patient has been diagnosed with a valvular disorder. The patient tells the nurse that he has read about numerous treatment options, including valvuloplasty. What should the nurse teach the patient about valvuloplasty? A) "For some patients, valvuloplasty can be done in a cardiac catheterization laboratory." B) "Valvuloplasty is a dangerous procedure, but it has excellent potential if it goes well." C) "Valvuloplasty is open heart surgery, but this is very safe these days and normally requires only an overnight hospital stay." D) "It's prudent to get a second opinion before deciding to have valvuloplasty."

Ans: "For some patients, valvuloplasty can be done in a cardiac catheterization laboratory." Feedback: Some valvuloplasty procedures do not require general anesthesia or cardiopulmonary bypass and can be performed in a cardiac catheterization laboratory or hybrid room. Open heart surgery is not required and the procedure does not carry exceptional risks that would designate it as being dangerous. Normally there is no need for the nurse to advocate for a second opinion.

A patient is a candidate for percutaneous balloon valvuloplasty, but is concerned about how this procedure will affect her busy work schedule. What guidance should the nurse provide to the patient? A) "Patients generally stay in the hospital for 6 to 8 days." B) "Patients are kept in the hospital until they are independent with all aspects of their care." C) "Patients need to stay in the hospital until they regain normal heart function for their age." D) "Patients usually remain at the hospital for 24 to 48 hours."

Ans: "Patients usually remain at the hospital for 24 to 48 hours." Feedback: After undergoing percutaneous balloon valvuloplasty, the patient usually remains in the hospital for 24 to 48 hours. Prediagnosis levels of heart function are not always attainable and the patient does not need to be wholly independent prior to discharge.

A patient is undergoing diagnostic testing for mitral stenosis. What statement by the patient during the nurse's interview is most suggestive of this valvular disorder? A) "I get chest pain from time to time, but it usually resolves when I rest." B) "Sometimes when I'm resting, I can feel my heart skip a beat." C) "Whenever I do any form of exercise I get terribly short of breath." D) "My feet and ankles have gotten terribly puffy the last few weeks."

Ans: "Whenever I do any form of exercise I get terribly short of breath." Feedback: The first symptom of mitral stenosis is often breathing difficulty (dyspnea) on exertion as a result of pulmonary venous hypertension. Patients with mitral stenosis are likely to show progressive fatigue as a result of low cardiac output. Palpitations occur in some patients, but dyspnea is a characteristic early symptom. Peripheral edema and chest pain are atypical.

10. The nurse has just admitted a 66-year-old patient for cardiac surgery. The patient tearfully admits to the nurse that she is afraid of dying while undergoing the surgery. What is the nurses best response? A) Explore the factors underlying the patients anxiety. B) Teach the patient guided imagery techniques. C) Obtain an order for a PRN benzodiazepine. D) Describe the procedure in greater detail.

Ans: A Feedback: An assessment of anxiety levels is required in the patient to assist the patient in identifying fears and developing coping mechanisms for those fears. The nurse must further assess and explore the patients anxiety before providing interventions such as education or medications.

15. The nurse is working with a patient who had an MI and is now active in rehabilitation. The nurse should teach this patient to cease activity if which of the following occurs? A) The patient experiences chest pain, palpitations, or dyspnea. B) The patient experiences a noticeable increase in heart rate during activity. C) The patients oxygen saturation level drops below 96%. D) The patients respiratory rate exceeds 30 breaths/min.

Ans: A Feedback: Any activity or exercise that causes dyspnea and chest pain should be stopped in the patient with CAD. Heart rate must not exceed the target rate, but an increase above resting rate is expected and is therapeutic. In most patients, a respiratory rate that exceeds 30 breaths/min is not problematic. Similarly, oxygen saturation slightly below 96% does not necessitate cessation of activity.

1. The nurse is caring for a patient who has been diagnosed with an elevated cholesterol level. The nurse is aware that plaque on the inner lumen of arteries is composed chiefly of what? A) Lipids and fibrous tissue B) White blood cells C) Lipoproteins D) High-density cholesterol

Ans: A Feedback: As T-lymphocytes and monocytes infiltrate to ingest lipids on the arterial wall and then die, a fibrous tissue develops. This causes plaques to form on the inner lumen of arterial walls. These plaques do not consist of white cells, lipoproteins, or high-density cholesterol.

16. A patient with cardiovascular disease is being treated with amlodipine (Norvasc), a calcium channel blocking agent. The therapeutic effects of calcium channel blockers include which of the following? A) Reducing the hearts workload by decreasing heart rate and myocardial contraction B) Preventing platelet aggregation and subsequent thrombosis C) Reducing myocardial oxygen consumption by blocking adrenergic stimulation to the heart D) Increasing the efficiency of myocardial oxygen consumption, thus decreasing ischemia and relieving pain

Ans: A Feedback: Calcium channel blocking agents decrease sinoatrial node automaticity and atrioventricular node conduction, resulting in a slower heart rate and a decrease in the strength of the heart muscle contraction. These effects decrease the workload of the heart. Antiplatelet and anticoagulation medications are administered to prevent platelet aggregation and subsequent thrombosis, which impedes blood flow. Beta-blockers reduce myocardial consumption by blocking beta-adrenergic sympathetic stimulation to the heart. The result is reduced myocardial contractility (force of contraction) to balance the myocardium oxygen needs and supply. Nitrates reduce myocardial oxygen consumption, which decreases ischemia and relieves pain by dilating the veins and, in higher doses, the arteries.

33. The nurse is caring for a patient who has undergone percutaneous transluminal coronary angioplasty (PTCA). What is the major indicator of success for this procedure? A) Increase in the size of the arterys lumen B) Decrease in arterial blood flow in relation to venous flow C) Increase in the patients resting heart rate D) Increase in the patients level of consciousness (LOC)

Ans: A Feedback: PTCA is used to open blocked coronary vessels and resolve ischemia. The procedure may result in beneficial changes to the patients LOC or heart rate, but these are not the overarching goals of PTCA. Increased arterial flow is the focus of the procedures.

18. When discussing angina pectoris secondary to atherosclerotic disease with a patient, the patient asks why he tends to experience chest pain when he exerts himself. The nurse should describe which of the following phenomena? A) Exercise increases the hearts oxygen demands. B) Exercise causes vasoconstriction of the coronary arteries. C) Exercise shunts blood flow from the heart to the mesenteric area. D) Exercise increases the metabolism of cardiac medications.

Ans: A Feedback: Physical exertion increases the myocardial oxygen demand. If the patient has arteriosclerosis of the coronary arteries, then blood supply is diminished to the myocardium. Exercise does not cause vasoconstriction or interfere with drug metabolism. Exercise does not shunt blood flow away from the heart.

8. The nurse is caring for a patient who is scheduled for cardiac surgery. What should the nurse include in preoperative care? A) With the patient, clarify the surgical procedure that will be performed. B) Withhold the patients scheduled medications for at least 12 hours preoperatively. C) Inform the patient that health teaching will begin as soon as possible after surgery. D) Avoid discussing the patients fears as not to exacerbate them.

Ans: A Feedback: Preoperatively, it is necessary to evaluate the patients understanding of the surgical procedure, informed consent, and adherence to treatment protocols. Teaching would begin on admission or even prior to admission. The physician would write orders to alter the patients medication regimen if necessary; this will vary from patient to patient. Fears should be addressed directly and empathically.

13. A 48-year-old man presents to the ED complaining of severe substernal chest pain radiating down his left arm. He is admitted to the coronary care unit (CCU) with a diagnosis of myocardial infarction (MI). What nursing assessment activity is a priority on admission to the CCU? A) Begin ECG monitoring. B) Obtain information about family history of heart disease. C) Auscultate lung fields. D) Determine if the patient smokes.

Ans: A Feedback: The 12-lead ECG provides information that assists in ruling out or diagnosing an acute MI. It should be obtained within 10 minutes from the time a patient reports pain or arrives in the ED. By monitoring serial ECG changes over time, the location, evolution, and resolution of an MI can be identified and monitored; life-threatening arrhythmias are the leading cause of death in the first hours after an MI. Obtaining information about family history of heart disease and whether the patient smokes are not immediate priorities in the acute phase of MI. Data may be obtained from family members later. Lung fields are auscultated after oxygenation and pain control needs are met.

27. A patient presents to the ED in distress and complaining of crushing chest pain. What is the nurses priority for assessment? A) Prompt initiation of an ECG B) Auscultation of the patients point of maximal impulse (PMI) C) Rapid assessment of the patients peripheral pulses D) Palpation of the patients cardiac apex

Ans: A Feedback: The 12-lead ECG provides information that assists in ruling out or diagnosing an acute MI. It should be obtained within 10 minutes from the time a patient reports pain or arrives in the ED. Each of the other listed assessments is valid, but ECG monitoring is the most time dependent priority.

4. The triage nurse in the ED assesses a 66-year-old male patient who presents to the ED with complaints of midsternal chest pain that has lasted for the last 5 hours. If the patients symptoms are due to an MI, what will have happened to the myocardium? A) It may have developed an increased area of infarction during the time without treatment. B) It will probably not have more damage than if he came in immediately. C) It may be responsive to restoration of the area of dead cells with proper treatment. D) It has been irreparably damaged, so immediate treatment is no longer necessary.

Ans: A Feedback: When the patient experiences lack of oxygen to myocardium cells during an MI, the sooner treatment is initiated, the more likely the treatment will prevent or minimize myocardial tissue necrosis. Delays in treatment equate with increased myocardial damage. Despite the length of time the symptoms have been present, treatment needs to be initiated immediately to minimize further damage. Dead cells cannot be restored by any means.

5. The nurse is conducting patient teaching about cholesterol levels. When discussing the patients elevated LDL and lowered HDL levels, the patient shows an understanding of the significance of these levels by stating what? A) Increased LDL and decreased HDL increase my risk of coronary artery disease. B) Increased LDL has the potential to decrease my risk of heart disease. C) The decreased HDL level will increase the amount of cholesterol moved away from the artery walls. D) The increased LDL will decrease the amount of cholesterol deposited on the artery walls.

Ans: A Feedback: Elevated LDL levels and decreased HDL levels are associated with a greater incidence of coronary artery disease.

16. The nurse working on a cardiac care unit is caring for a patient whose stroke volume has increased. The nurse is aware that afterload influences a patients stroke volume. The nurse recognizes that afterload is increased when there is what? A) Arterial vasoconstriction B) Venous vasoconstriction C) Arterial vasodilation D) Venous vasodilation

Ans: A Feedback: Arterial vasoconstriction increases the systemic vascular resistance, which increases the afterload. Venous vasoconstriction decreases preload thereby decreasing stroke volume. Venous vasodilation increases preload.

6. The physician has placed a central venous pressure (CVP) monitoring line in an acutely ill patient so right ventricular function and venous blood return can be closely monitored. The results show decreased CVP. What does this indicate? A) Possible hypovolemia B) Possible myocardial infarction (MI) C) Left-sided heart failure D) Aortic valve regurgitation

Ans: A Feedback: Hypovolemia may cause a decreased CVP. MI, valve regurgitation and heart failure are less likely causes of decreased CVP.

13. The critical care nurse is caring for a patient with a pulmonary artery pressure monitoring system. The nurse is aware that pulmonary artery pressure monitoring is used to assess left ventricular function. What is an additional function of pulmonary artery pressure monitoring systems? A) To assess the patients response to fluid and drug administration B) To obtain specimens for arterial blood gas measurements C) To dislodge pulmonary emboli D) To diagnose the etiology of chronic obstructive pulmonary disease

Ans: A Feedback: Pulmonary artery pressure monitoring is an important tool used in critical care for assessing left ventricular function (cardiac output), diagnosing the etiology of shock, and evaluating the patients response to medical interventions, such as fluid administration and vasoactive medications. Pulmonary artery monitoring is preferred for the patient with heart failure over central venous pressure monitoring. Arterial catheters are useful when arterial blood gas measurements and blood samples need to be obtained frequently. Neither intervention is used to clear pulmonary emboli.

3. The nurse is calculating a cardiac patients pulse pressure. If the patients blood pressure is 122/76 mm Hg, what is the patients pulse pressure? A) 46 mmHg B) 99 mmHg C) 198 mmHg D) 76 mmHg

Ans: A Feedback: Pulse pressure is the difference between the systolic and diastolic pressure. In this case, this value is 46 mm Hg.

1. A nurse is describing the process by which blood is ejected into circulation as the chambers of the heart become smaller. The instructor categorizes this action of the heart as what? A) Systole B) Diastole C) Repolarization D) Ejection fraction

Ans: A Feedback: Systole is the action of the chambers of the heart becoming smaller and ejecting blood. This action of the heart is not diastole (relaxations), ejection fraction (the amount of blood expelled), or repolarization (electrical charging).

29. The nurse is assessing a patient with acute coronary syndrome (ACS). The nurse includes a careful history in the assessment, especially with regard to signs and symptoms. What signs and symptoms are suggestive of ACS? Select all that apply. A) Dyspnea B) Unusual fatigue C) Hypotension D) Syncope E) Peripheral cyanosis

Ans: A, B, D Feedback: Systematic assessment includes a careful history, particularly as it relates to symptoms: chest pain or discomfort, difficulty breathing (dyspnea), palpitations, unusual fatigue, faintness (syncope), or sweating (diaphoresis). Each symptom must be evaluated with regard to time, duration, and the factors that precipitate the symptom and relieve it, and in comparison with previous symptoms. Hypotension and peripheral cyanosis are not typically associated with ACS.

38. The nurse providing care for a patient post PTCA knows to monitor the patient closely. For what complications should the nurse monitor the patient? Select all that apply. A) Abrupt closure of the coronary artery B) Venous insufficiency C) Bleeding at the insertion site D) Retroperitoneal bleeding E) Arterial occlusion

Ans: A, C, D, E Feedback: Complications after the procedure may include abrupt closure of the coronary artery and vascular complications, such as bleeding at the insertion site, retroperitoneal bleeding, hematoma, and arterial occlusion, as well as acute renal failure. Venous insufficiency is not a postprocedure complication of a PTCA.

A patient with pericarditis has just been admitted to the CCU. The nurse planning the patient's care should prioritize what nursing diagnosis? A) Anxiety related to pericarditis B) Acute pain related to pericarditis C) Ineffective tissue perfusion related to pericarditis D) Ineffective breathing pattern related to pericarditis

Ans: Acute pain related to pericarditis Feedback: The most characteristic symptom of pericarditis is chest pain, although pain also may be located beneath the clavicle, in the neck, or in the left trapezius (scapula) region. The pain or discomfort usually remains fairly constant, but it may worsen with deep inspiration and when lying down or turning. Anxiety is highly plausible and should be addressed, but chest pain is a nearly certain accompaniment to the disease. Breathing and tissue perfusion are likely to be at risk, but pain is certain, especially in the early stages of treatment.

A patient with a history rheumatic heart disease knows that she is at risk for bacterial endocarditis when undergoing invasive procedures. Prior to a scheduled cystoscopy, the nurse should ensure that the patient knows the importance of taking which of the following drugs? A) Enoxaparin (Lovenox) B) Metoprolol (Lopressor) C) Azathioprine (Imuran) D) Amoxicillin (Amoxil)

Ans: Amoxicillin (Amoxil) Feedback: Although rare, bacterial endocarditis may be life-threatening. A key strategy is primary prevention in high-risk patients (i.e., those with rheumatic heart disease, mitral valve prolapse, or prosthetic heart valves). Antibiotic prophylaxis is recommended for high-risk patients immediately before and sometimes after certain procedures. Amoxicillin is the drug of choice. None of the other listed drugs is an antibiotic.

Most individuals who have mitral valve prolapse never have any symptoms, although this is not the case for every patient. What symptoms might a patient have with mitral valve prolapse? Select all that apply. A) Anxiety B) Fatigue C) Shoulder pain D) Tachypnea E) Palpitations

Ans: Anxiety, Fatigue, Palpitations Feedback: Most people who have mitral valve prolapse never have symptoms. A few have symptoms of fatigue, shortness of breath, lightheadedness, dizziness, syncope, palpitations, chest pain, and anxiety. Hyperpnea and shoulder pain are not characteristic symptoms of mitral valve prolapse.

A patient with mitral stenosis exhibits new symptoms of a dysrhythmia. Based on the pathophysiology of this disease process, the nurse would expect the patient to exhibit what heart rhythm? A) Ventricular fibrillation (VF) B) Ventricular tachycardia (VT) C) Atrial fibrillation D) Sinus bradycardia

Ans: Atrial fibrillation Feedback: In patients with mitral valve stenosis, the pulse is weak and often irregular because of atrial fibrillation. Bradycardia, VF, and VT are not characteristic of this valvular disorder.

A patient with mitral valve stenosis is receiving health education at an outpatient clinic. To minimize the patient's symptoms, the nurse should teach the patient to do which of the following? A) Eat a high-protein, low-carbohydrate diet. B) Avoid activities that cause an increased heart rate. C) Avoid large crowds and public events. D) Perform deep breathing and coughing exercises.

Ans: Avoid activities that cause an increased heart rate. Feedback: Patients with mitral stenosis are advised to avoid strenuous activities, competitive sports, and pregnancy, all of which increase heart rate. Infection prevention is important, but avoiding crowds is not usually necessary. Deep breathing and coughing are not likely to prevent exacerbations of symptoms and increased protein intake is not necessary.

The critical care nurse is caring for a patient who is receiving cyclosporine postoperative heart transplant. The patient asks the nurse to remind him what this medication is for. How should the nurse best respond? A) Azathioprine decreases the risk of thrombus formation. B) Azathioprine ensures adequate cardiac output. C) Azathioprine increases the number of white blood cells. D) Azathioprine minimizes rejection of the transplant.

Ans: Azathioprine minimizes rejection of the transplant. Feedback: After heart transplant, patients are constantly balancing the risk of rejection with the risk of infection. Most commonly, patients receive cyclosporine or tacrolimus (FK506, Prograf), azathioprine (Imuran), or mycophenolate mofetil (CellCept), and corticosteroids (prednisone) to minimize rejection. Cyclosporine does not prevent thrombus formation, enhance cardiac output, or increase white cell counts.

34. A nurse has taken on the care of a patient who had a coronary artery stent placed yesterday. When reviewing the patients daily medication administration record, the nurse should anticipate administering what drug? A) Ibuprofen B) Clopidogrel C) Dipyridamole D) Acetaminophen

Ans: B Feedback: Because of the risk of thrombus formation within the stent, the patient receives antiplatelet medications, usually aspirin and clopidogrel. Ibuprofen and acetaminophen are not antiplatelet drugs. Dipyridamole is not the drug of choice following stent placement.

11. A patient with angina has been prescribed nitroglycerin. Before administering the drug, the nurse should inform the patient about what potential adverse effects? A) Nervousness or paresthesia B) Throbbing headache or dizziness C) Drowsiness or blurred vision D) Tinnitus or diplopia

Ans: B Feedback: Headache and dizziness commonly occur when nitroglycerin is taken at the beginning of therapy. Nervousness, paresthesia, drowsiness, blurred vision, tinnitus, and diplopia do not typically occur as a result of nitroglycerin therapy.

30. The nurse is creating a plan of care for a patient with acute coronary syndrome. What nursing action should be included in the patients care plan? A) Facilitate daily arterial blood gas (ABG) sampling. B) Administer supplementary oxygen, as needed. C) Have patient maintain supine positioning when in bed. D) Perform chest physiotherapy, as indicated.

Ans: B Feedback: Oxygen should be administered along with medication therapy to assist with symptom relief. Administration of oxygen raises the circulating level of oxygen to reduce pain associated with low levels of myocardial oxygen. Physical rest in bed with the head of the bed elevated or in a supportive chair helps decrease chest discomfort and dyspnea. ABGs are diagnostic, not therapeutic, and they are rarely needed on a daily basis. Chest physiotherapy is not used in the treatment of ACS.

19. The nurse is caring for a patient who is believed to have just experienced an MI. The nurse notes changes in the ECG of the patient. What change on an ECG most strongly suggests to the nurse that ischemia is occurring? A) P wave inversion B) T wave inversion C) Q wave changes with no change in ST or T wave D) P wave enlargement

Ans: B Feedback: T-wave inversion is an indicator of ischemic damage to myocardium. Typically, few changes to P waves occur during or after an MI, whereas Q-wave changes with no change in the ST or T wave indicate an old MI.

24. When assessing a patient diagnosed with angina pectoris it is most important for the nurse to gather what information? A) The patients activities limitations and level of consciousness after the attacks B) The patients symptoms and the activities that precipitate attacks C) The patients understanding of the pathology of angina D) The patients coping strategies surrounding the attacks

Ans: B Feedback: The nurse must gather information about the patients symptoms and activities, especially those that precede and precipitate attacks of angina pectoris. The patients coping, understanding of the disease, and status following attacks are all important to know, but causative factors are a primary focus of the assessment interview.

20. An adult patient is admitted to the ED with chest pain. The patient states that he had developed unrelieved chest pain that was present for approximately 20 minutes before coming to the hospital. To minimize cardiac damage, the nurse should expect to administer which of the following interventions? A) Thrombolytics, oxygen administration, and nonsteroidal anti-inflammatories B) Morphine sulphate, oxygen, and bed rest C) Oxygen and beta-adrenergic blockers D) Bed rest, albuterol nebulizer treatments, and oxygen

Ans: B Feedback: The patient with suspected MI should immediately receive supplemental oxygen, aspirin, nitroglycerin, and morphine. Morphine sulphate reduces preload and decreases workload of the heart, along with increased oxygen from oxygen therapy and bed rest. With decreased cardiac demand, this provides the best chance of decreasing cardiac damage. NSAIDs and beta-blockers are not normally indicated. Albuterol, which is a medication used to manage asthma and respiratory conditions, will increase the heart rate.

22. In preparation for cardiac surgery, a patient was taught about measures to prevent venous thromboembolism. What statement indicates that the patient clearly understood this education? A) Ill try to stay in bed for the first few days to allow myself to heal. B) Ill make sure that I dont cross my legs when Im resting in bed. C) Ill keep pillows under my knees to help my blood circulate better. D) Ill put on those compression stockings if I get pain in my calves.

Ans: B Feedback: To prevent venous thromboembolism, patients should avoid crossing the legs. Activity is generally begun as soon as possible and pillows should not be placed under the popliteal space. Compression stockings are often used to prevent venous thromboembolism, but they would not be applied when symptoms emerge.

7. A patient with an occluded coronary artery is admitted and has an emergency percutaneous transluminal coronary angioplasty (PTCA). The patient is admitted to the cardiac critical care unit after the PTCA. For what complication should the nurse most closely monitor the patient? A) Hyperlipidemia B) Bleeding at insertion site C) Left ventricular hypertrophy D) Congestive heart failure

Ans: B Test Bank - Brunner & Suddarth's Textbook of Medical-Surgical Nursing 14e (Hinkle 2017) 529 Feedback: Complications of PTCA may include bleeding at the insertion site, abrupt closure of the artery, arterial thrombosis, and perforation of the artery. Complications do not include hyperlipidemia, left ventricular hypertrophy, or congestive heart failure; each of these problems takes an extended time to develop and none is emergent.

12. The critical care nurse is caring for a patient with a central venous pressure (CVP) monitoring system. The nurse notes that the patients CVP is increasing. Of what may this indicate? A) Psychosocial stress B) Hypervolemia C) Dislodgment of the catheter D) Hypomagnesemia

Ans: B Feedback: CVP is a useful hemodynamic parameter to observe when managing an unstable patients fluid volume status. An increasing pressure may be caused by hypervolemia or by a condition, such as heart failure, that results in decreased myocardial contractility. Stress, dislodgement of the catheter, and low magnesium levels would not typically result in increased CVP.

9. The nurse is caring for a patient who has a history of heart disease. What factor should the nurse identify as possibly contributing to a decrease in cardiac output? A) A change in position from standing to sitting B) A heart rate of 54 bpm C) A pulse oximetry reading of 94% D) An increase in preload related to ambulation

Ans: B Feedback: Cardiac output is computed by multiplying the stroke volume by the heart rate. Cardiac output can be affected by changes in either stroke volume or heart rate, such as a rate of 54 bpm. An increase in preload will lead to an increase in stroke volume. A pulse oximetry reading of 94% does not indicate hypoxemia, as hypoxia can decrease contractility. Transitioning from standing to sitting would more likely increase rather than decrease cardiac output.

11. A resident of a long-term care facility has complained to the nurse of chest pain. What aspect of the residents pain would be most suggestive of angina as the cause? A) The pain is worse when the resident inhales deeply. B) The pain occurs immediately following physical exertion. C) The pain is worse when the resident coughs. D) The pain is most severe when the resident moves his upper body.

Ans: B Feedback: Chest pain associated with angina is often precipitated by physical exertion. The other listed aspects of chest pain are more closely associated with noncardiac etiologies.

19. The critical care nurse is caring for a patient who has had an MI. The nurse should expect to assist with establishing what hemodynamic monitoring procedure to assess the patients left ventricular function? A) Central venous pressure (CVP) monitoring B) Pulmonary artery pressure monitoring (PAPM) C) Systemic arterial pressure monitoring (SAPM) D) Arterial blood gases (ABG)

Ans: B Feedback: PAPM is used to assess left ventricular function. CVP is used to assess right ventricular function; SAPM is used for continual assessment of BP. ABG are used to assess for acidic and alkalotic levels in the blood.

7. While auscultating a patients heart sounds, the nurse hears an extra heart sound immediately after the second heart sound (S2). An audible S3 would be considered an expected finding in what patient? A) An older adult B) A 20-year-old patient C) A patient who has undergone valve replacement D) A patient who takes a beta-adrenergic blocker

Ans: B Feedback: S3 represents a normal finding in children and adults up to 35 or 40 years of age. In these cases, it is called a physiologic S3 . It is an abnormal finding in a patient with an artificial valve, an older adult, or a patient who takes a beta blocker.

A patient has been admitted to the medical unit with signs and symptoms suggestive of endocarditis. The physician's choice of antibiotics would be primarily based on what diagnostic test? A) Echocardiography B) Blood cultures C) Cardiac aspiration D) Complete blood count

Ans: Blood cultures Feedback: To help determine the causative organisms and the most effective antibiotic treatment for the patient, blood cultures are taken. A CBC can help establish the degree and stage of infection, but not the causative microorganism. Echocardiography cannot indicate the microorganisms causing the infection. "Cardiac aspiration" is not a diagnostic test.

An older adult patient has been diagnosed with aortic regurgitation. What change in blood flow should the nurse expect to see on this patient's echocardiogram? A) Blood to flow back from the aorta to the left ventricle B) Obstruction of blood flow from the left ventricle C) Blood to flow back from the left atrium to the left ventricle D) Obstruction of blood from the left atrium to left ventricle

Ans: Blood to flow back from the aorta to the left ventricle Feedback: Aortic regurgitation occurs when the aortic valve does not completely close, and blood flows back to the left ventricle from the aorta during diastole. Aortic regurgitation does not cause obstruction of blood flow from the left ventricle, blood to flow back from the left atrium to the left ventricle, or obstruction of blood from the left atrium to left ventricle.

35. A nurse is working with a patient who has been scheduled for a percutaneous coronary intervention (PCI) later in the week. What anticipatory guidance should the nurse provide to the patient? A) He will remain on bed rest for 48 to 72 hours after the procedure. B) He will be given vitamin K infusions to prevent bleeding following PCI. C) A sheath will be placed over the insertion site after the procedure is finished. D) The procedure will likely be repeated in 6 to 8 weeks to ensure success.

Ans: C Feedback: A sheath is placed over the PCI access site and kept in place until adequate coagulation is achieved. Patients resume activity a few hours after PCI and repeated treatments may or may not be necessary. Anticoagulants, not vitamin K, are administered during PCI.

25. You are writing a care plan for a patient who has been diagnosed with angina pectoris. The patient describes herself as being distressed and shocked by her new diagnosis. What nursing diagnosis is most clearly suggested by the womans statement? A) Spiritual distress related to change in health status B) Acute confusion related to prognosis for recovery C) Anxiety related to cardiac symptoms D) Deficient knowledge related to treatment of angina pectoris

Ans: C Feedback: Although further assessment is warranted, it is not unlikely that the patient is experiencing anxiety. In patients with CAD, this often relates to the threat of sudden death. There is no evidence of confusion (i.e., delirium or dementia) and there may or may not be a spiritual element to her concerns. Similarly, it is not clear that a lack of knowledge or information is the root of her anxiety.

5. Family members bring a patient to the ED with pale cool skin, sudden midsternal chest pain unrelieved with rest, and a history of CAD. How should the nurse best interpret these initial data? A) The symptoms indicate angina and should be treated as such. B) The symptoms indicate a pulmonary etiology rather than a cardiac etiology. C) The symptoms indicate an acute coronary episode and should be treated as such. D) Treatment should be determined pending the results of an exercise stress test.

Ans: C Feedback: Angina and MI have similar symptoms and are considered the same process, but are on different points along a continuum. That the patients symptoms are unrelieved by rest suggests an acute coronary episode rather than angina. Pale cool skin and sudden onset are inconsistent with a pulmonary etiology. Treatment should be initiated immediately regardless of diagnosis.

31. The nurse is participating in the care conference for a patient with ACS. What goal should guide the care teams selection of assessments, interventions, and treatments? A) Maximizing cardiac output while minimizing heart rate B) Decreasing energy expenditure of the myocardium C) Balancing myocardial oxygen supply with demand D) Increasing the size of the myocardial muscle

Ans: C Feedback: Balancing myocardial oxygen supply with demand (e.g., as evidenced by the relief of chest pain) is the top priority in the care of the patient with ACS. Treatment is not aimed directly at minimizing heart rate because some patients experience bradycardia. Increasing the size of the myocardium is never a goal. Reducing the myocardiums energy expenditure is often beneficial, but this must be balanced with productivity.

40. A patient is recovering in the hospital from cardiac surgery. The nurse has identified the diagnosis of risk for ineffective airway clearance related to pulmonary secretions. What intervention best addresses this risk? A) Administration of bronchodilators by nebulizer B) Administration of inhaled corticosteroids by metered dose inhaler (MDI) C) Patients consistent performance of deep breathing and coughing exercises D) Patients active participation in the cardiac rehabilitation program

Ans: C Feedback: Clearance of pulmonary secretions is accomplished by frequent repositioning of the patient, suctioning, and chest physical therapy, as well as educating and encouraging the patient to breathe deeply and cough. Medications are not normally used to achieve this goal. Rehabilitation is important, but will not necessarily aid the mobilization of respiratory secretions.

21. The nurse is assessing a patient who was admitted to the critical care unit 3 hours ago following cardiac surgery. The nurses most recent assessment reveals that the patients left pedal pulses are not palpable and that the right pedal pulses are rated at +2. What is the nurses best response? A) Document this expected assessment finding during the initial postoperative period. B) Reposition the patient with his left leg in a dependent position. C) Inform the patients physician of this assessment finding. D) Administer an ordered dose of subcutaneous heparin.

Ans: C Feedback: If a pulse is absent in any extremity, the cause may be prior catheterization of that extremity, chronic peripheral vascular disease, or a thromboembolic obstruction. The nurse immediately reports newly identified absence of any pulse.

3. The nurse is caring for an adult patient who had symptoms of unstable angina upon admission to the hospital. What nursing diagnosis underlies the discomfort associated with angina? A) Ineffective breathing pattern related to decreased cardiac output B) Anxiety related to fear of death C) Ineffective cardiopulmonary tissue perfusion related to coronary artery disease (CAD) D) Impaired skin integrity related to CAD

Ans: C Feedback: Ineffective cardiopulmonary tissue perfusion directly results in the symptoms of discomfort associated with angina. Anxiety and ineffective breathing may result from angina chest pain, but they are not the causes. Skin integrity is not impaired by the effects of angina.

39. A patient who is postoperative day 1 following a CABG has produced 20 mL of urine in the past 3 hours and the nurse has confirmed the patency of the urinary catheter. What is the nurses most appropriate action? A) Document the patients low urine output and monitor closely for the next several hours. B) Contact the dietitian and suggest the need for increased oral fluid intake. C) Contact the patients physician and suggest assessment of fluid balance and renal function. D) Increase the infusion rate of the patients IV fluid to prompt an increase in renal function.

Ans: C Feedback: Nursing management includes accurate measurement of urine output. An output of less than 1 mL/kg/h may indicate hypovolemia or renal insufficiency. Prompt referral is necessary. IV fluid replacement may be indicated, but is beyond the independent scope of the dietitian or nurse.

28. The ED nurse is caring for a patient with a suspected MI. What drug should the nurse anticipate administering to this patient? A) Oxycodone B) Warfarin C) Morphine D) Acetaminophen

Ans: C Feedback: The patient with suspected MI is given aspirin, nitroglycerin, morphine, an IV beta- blocker, and other medications, as indicated, while the diagnosis is being confirmed. Tylenol, warfarin, and oxycodone are not typically used.

12. The nurse is providing an educational workshop about coronary artery disease (CAD) and its risk factors. The nurse explains to participants that CAD has many risk factors, some that can be controlled and some that cannot. What risk factors would the nurse list that can be controlled or modified? A) Gender, obesity, family history, and smoking B) Inactivity, stress, gender, and smoking C) Obesity, inactivity, diet, and smoking D) Stress, family history, and obesity

Ans: C Feedback: The risk factors for CAD that can be controlled or modified include obesity, inactivity, diet, stress, and smoking. Gender and family history are risk factors that cannot be controlled.

23. The nurse is performing an intake assessment on a patient with a new diagnosis of coronary artery disease. What would be the most important determination to make during this intake assessment? A) Whether the patient and involved family members understand the role of genetics in the etiology of the disease B) Whether the patient and involved family members understand dietary changes and the role of nutrition C) Whether the patient and involved family members are able to recognize symptoms of an acute cardiac problem and respond appropriately D) Whether the patient and involved family members understand the importance of social support and community agencies

Ans: C Feedback: During the health history, the nurse needs to determine if the patient and involved family members are able to recognize symptoms of an acute cardiac problem, such as acute coronary syndrome (ACS) or HF, and seek timely treatment for these symptoms. Each of the other listed topics is valid, but the timely and appropriate response to a cardiac emergency is paramount.

17. A nurse is preparing a patient for scheduled transesophageal echocardiography. What action should the nurse perform? A) Instruct the patient to drink 1 liter of water before the test. B) Administer IV benzodiazepines and opioids. C) Inform the patient that she will remain on bed rest following the procedure. D) Inform the patient that an access line will be initiated in her femoral artery.

Ans: C Feedback: During the recovery period, the patient must maintain bed rest with the head of the bed elevated to 45 degrees. The patient must be NPO 6 hours preprocedure. The patient is sedated to make him or her comfortable, but will not be heavily sedated, and opioids are not necessary. Also, the patient will have a peripheral IV line initiated preprocedure.

4. The nurse is caring for a patient admitted with unstable angina. The laboratory result for the initial troponin I is elevated in this patient. The nurse should recognize what implication of this assessment finding? A) This is only an accurate indicator of myocardial damage when it reaches its peak in 24 hours. B) Because the patient has a history of unstable angina, this is a poor indicator of myocardial injury. C) This is an accurate indicator of myocardial injury. D) This result indicates muscle injury, but does not specify the source.

Ans: C Feedback: Troponin I, which is specific to cardiac muscle, is elevated within hours after myocardial injury. Even with a diagnosis of unstable angina, this is an accurate indicator of myocardial injury.

32. The nurse working on the coronary care unit is caring for a patient with ACS. How can the nurse best meet the patients psychosocial needs? A) Reinforce the fact that treatment will be successful. B) Facilitate a referral to a chaplain or spiritual leader. C) Increase the patients participation in rehabilitation activities. D) Directly address the patients anxieties and fears.

Ans: D Feedback: Alleviating anxiety and decreasing fear are important nursing functions that reduce the sympathetic stress response. Referrals to spiritual care may or may not be appropriate, and this does not relieve the nurse of responsibility for addressing the patients psychosocial needs. Treatment is not always successful, and false hope should never be fostered. Participation in rehabilitation may alleviate anxiety for some patients, but it may exacerbate it for others.

9. The OR nurse is explaining to a patient that cardiac surgery requires the absence of blood from the surgical field. At the same time, it is imperative to maintain perfusion of body organs and tissues. What technique for achieving these simultaneous goals should the nurse describe? A) Coronary artery bypass graft (CABG) B) Percutaneous transluminal coronary angioplasty (PTCA) C) Atherectomy D) Cardiopulmonary bypass

Ans: D Feedback: Cardiopulmonary bypass is often used to circulate and oxygenate blood mechanically while bypassing the heart and lungs. PTCA, atherectomy, and CABG are all surgical procedures, none of which achieves the two goals listed.

14. The public health nurse is participating in a health fair and interviews a patient with a history of hypertension, who is currently smoking one pack of cigarettes per day. She denies any of the most common manifestations of CAD. Based on these data, the nurse would expect the focuses of CAD treatment most likely to be which of the following? A) Drug therapy and smoking cessation B) Diet and drug therapy C) Diet therapy only D) Diet therapy and smoking cessation

Ans: D Feedback: Due to the absence of symptoms, dietary therapy would likely be selected as the first-line treatment for possible CAD. Drug therapy would be determined based on a number of considerations and diagnostics findings, but would not be directly indicated. Smoking cessation is always indicated, regardless of the presence or absence of symptoms.

2. A patient presents to the walk-in clinic complaining of intermittent chest pain on exertion, which is eventually attributed to angina. The nurse should inform the patient that angina is most often attributable to what cause? A) Decreased cardiac output B) Decreased cardiac contractility C) Infarction of the myocardium D) Coronary arteriosclerosis

Ans: D Feedback: In most cases, angina pectoris is due to arteriosclerosis. The disease is not a result of impaired cardiac output or contractility. Infarction may result from untreated angina, but it is not a cause of the disease.

36. Preoperative education is an important part of the nursing care of patients having coronary artery revascularization. When explaining the pre- and postoperative regimens, the nurse would be sure to include education about which subject? A) Symptoms of hypovolemia B) Symptoms of low blood pressure C) Complications requiring graft removal D) Intubation and mechanical ventilation

Ans: D Feedback: Most patients remain intubated and on mechanical ventilation for several hours after surgery. It is important that patients realize that this will prevent them from talking, and the nurse should reassure them that the staff will be able to assist them with other means of communication. Teaching would generally not include symptoms of low blood pressure or hypovolemia, as these are not applicable to most patients. Teaching would also generally not include rare complications that would require graft removal.

17. The nurse is providing care for a patient with high cholesterol and triglyceride values. In teaching the patient about therapeutic lifestyle changes such as diet and exercise, the nurse realizes that the desired goal for cholesterol levels is which of the following? A) High HDL values and high triglyceride values B) Absence of detectable total cholesterol levels C) Elevated blood lipids, fasting glucose less than 100 D) Low LDL values and high HDL values

Ans: D Feedback: The desired goal for cholesterol readings is for a patient to have low LDL and high HDL values. LDL exerts a harmful effect on the coronary vasculature because the small LDL particles can be easily transported into the vessel lining. In contrast, HDL promotes the use of total cholesterol by transporting LDL to the liver, where it is excreted. Elevated triglycerides are also a major risk factor for cardiovascular disease. A goal is also to keep triglyceride levels less than 150 mg/dL. All individuals possess detectable levels of total cholesterol.

37. A patient in the cardiac step-down unit has begun bleeding from the percutaneous coronary intervention (PCI) access site in her femoral region. What is the nurses most appropriate action? A) Call for assistance and initiate cardiopulmonary resuscitation. B) Reposition the patients leg in a nondependent position. C) Promptly remove the femoral sheath. D) Call for help and apply pressure to the access site.

Ans: D Feedback: The femoral sheath produces pressure on the access site. Pressure will temporarily reduce bleeding and allow for subsequent interventions. Removing the sheath would exacerbate bleeding and repositioning would not halt it. CPR is not indicated unless there is evidence of respiratory or cardiac arrest.

6. An OR nurse is preparing to assist with a coronary artery bypass graft (CABG). The OR nurse knows that the vessel most commonly used as source for a CABG is what? A) Brachial artery B) Brachial vein C) Femoral artery D) Greater saphenous vein

Ans: D Feedback: The greater saphenous vein is the most commonly used graft site for CABG. The right and left internal mammary arteries, radial arteries, and gastroepiploic artery are other graft sites used, though not as frequently. The femoral artery, brachial artery, and brachial vein are never harvested.

26. The nurse is caring for patient who tells the nurse that he has an angina attack beginning. What is the nurses most appropriate initial action? A) Have the patient sit down and put his head between his knees. B) Have the patient perform pursed-lip breathing. C) Have the patient stand still and bend over at the waist. D) Place the patient on bed rest in a semi-Fowlers position.

Ans: D Feedback: When a patient experiences angina, the patient is directed to stop all activities and sit or rest in bed in a semi-Fowlers position to reduce the oxygen requirements of the ischemic myocardium. Pursed-lip breathing and standing will not reduce workload to the same extent. No need to have the patient put his head between his legs because cerebral perfusion is not lacking.

10. The nurse is caring for an 82-year-old patient. The nurse knows that changes in cardiac structure and function occur in older adults. What is a normal change expected in the aging heart of an older adult? A) Decreased left ventricular ejection time B) Decreased connective tissue in the SA and AV nodes and bundle branches C) Thinning and flaccidity of the cardiac values D) Widening of the aorta

Ans: D Feedback: Changes in cardiac structure and function are clearly observable in the aging heart. Aging results in decreased elasticity and widening of the aorta, thickening and rigidity of the cardiac valves, increased connective tissue in the SA and AV nodes and bundle branches, and an increased left ventricular ejection time (prolonged systole).

22. A patient is brought into the ED by family members who tell the nurse the patient grabbed his chest and complained of substernal chest pain. The care team recognizes the need to monitor the patients cardiac function closely while interventions are performed. What form of monitoring should the nurse anticipate? A) Left-sided heart catheterization B) Cardiac telemetry C) Transesophageal echocardiography D) Hardwire continuous ECG monitoring

Ans: D Feedback: Two types of continuous ECG monitoring techniques are used in health care settings: hardwire cardiac monitoring, found in EDs, critical care units, and progressive care units; and telemetry, found in general nursing care units or outpatient cardiac rehabilitation programs. Cardiac catheterization and transesophageal echocardiography would not be used in emergent situations to monitor cardiac function.

23. An ED nurse is assessing an adult woman for a suspected MI. When planning the assessment, the nurse should be cognizant of what signs and symptoms of MI that are particularly common in female patients? Select all that apply. A) Shortness of breath B) Chest pain C) Anxiety D) Numbness E) Weakness

Ans: D, E Feedback: Although these symptoms are not wholly absent in men, many women have been found to have atypical symptoms of MI, including indigestion, nausea, palpitations, and numbness. Shortness of breath, chest pain, and anxiety are common symptoms of MI among patients of all ages and genders.

The cardiac nurse is caring for a patient who has been diagnosed with dilated cardiomyopathy (DCM). Echocardiography is likely to reveal what pathophysiological finding? A) Decreased ejection fraction B) Decreased heart rate C) Ventricular hypertrophy D) Mitral valve regurgitation

Ans: Decreased ejection fraction Feedback: DCM is distinguished by significant dilation of the ventricles without simultaneous hypertrophy. The ventricles have elevated systolic and diastolic volumes, but a decreased ejection fraction. Bradycardia and mitral valve regurgitation do not typically occur in patients with DCM.

A patient who has undergone a valve replacement with a mechanical valve prosthesis is due to be discharged home. During discharge teaching, the nurse should discuss the importance of antibiotic prophylaxis prior to which of the following? A) Exposure to immunocompromised individuals B) Future hospital admissions C) Dental procedures D) Live vaccinations

Ans: Dental procedures Feedback: Following mechanical valve replacement, antibiotic prophylaxis is necessary before dental procedures involving manipulation of gingival tissue, the periapical area of the teeth or perforation of the oral mucosa (not including routine anesthetic injections, placement of orthodontic brackets, or loss of deciduous teeth). There are no current recommendations around antibiotic prophylaxis prior to vaccination, future hospital admissions, or exposure to people who are immunosuppressed.

The nurse is reviewing the echocardiography results of a patient who has just been diagnosed with dilated cardiomyopathy (DCM). What changes in heart structure characterize DCM? A) Dilated ventricles with atrophy of the ventricles B) Dilated ventricles without hypertrophy of the ventricles C) Dilation and hypertrophy of all four heart chambers D) Dilation of the atria and hypertrophy of the ventricles

Ans: Dilated ventricles without hypertrophy of the ventricles Feedback: DCM is characterized by significant dilation of the ventricles without significant concomitant hypertrophy and systolic dysfunction. The ventricles do not atrophy in patients with DCM.

The patient has just returned to the floor after balloon valvuloplasty of the aortic valve and the nurse is planning appropriate assessments. The nurse should know that complications following this procedure include what? Select all that apply. A) Emboli B) Mitral valve damage C) Ventricular dysrhythmia D) Atrial-septal defect E) Plaque formation

Ans: Emboli, Mitral valve damage, Ventricular dysrhythmia Feedback: Possible complications include aortic regurgitation, emboli, ventricular perforation, rupture of the aortic valve annulus, ventricular dysrhythmia, mitral valve damage, and bleeding from the catheter insertion sites. Atrial-septal defect and plaque formation are not complications of a balloon valvuloplasty.

A patient who has recently recovered from a systemic viral infection is undergoing diagnostic testing for myocarditis. Which of the nurse's assessment findings is most consistent with myocarditis? A) Sudden changes in level of consciousness (LOC) B) Peripheral edema and pulmonary edema C) Pleuritic chest pain D) Flulike symptoms

Ans: Flulike symptoms Feedback: The most common symptoms of myocarditis are flulike. Chest pain, edema, and changes in LOC are not characteristic of myocarditis.

The nurse is auscultating the breath sounds of a patient with pericarditis. What finding is most consistent with this diagnosis? A) Wheezes B) Friction rub C) Fine crackles D) Coarse crackles

Ans: Friction rub Feedback: A pericardial friction rub is diagnostic of pericarditis. Crackles are associated with pulmonary edema and fluid accumulation, whereas wheezes signal airway constriction; neither of these occurs with pericarditis.

A patient has been living with dilated cardiomyopathy for several years but has experienced worsening symptoms despite aggressive medical management. The nurse should anticipate what potential treatment? A) Heart transplantation B) Balloon valvuloplasty C) Cardiac catheterization D) Stent placement

Ans: Heart transplantation Feedback: When heart failure progresses and medical treatment is no longer effective, surgical intervention, including heart transplantation, is considered. Valvuloplasty, stent placement, and cardiac catheterization will not address the pathophysiology of cardiomyopathy.

The nurse on the hospital's infection control committee is looking into two cases of hospital-acquired infective endocarditis among a specific classification of patients. What classification of patients would be at greatest risk for hospital-acquired endocarditis? A) Hemodialysis patients B) Patients on immunoglobulins C) Patients who undergo intermittent urinary catheterization D) Children under the age of 12

Ans: Hemodialysis patients Feedback: Hospital-acquired infective endocarditis occurs most often in patients with debilitating disease or indwelling catheters and in patients who are receiving hemodialysis or prolonged IV fluid or antibiotic therapy. Patients taking immunosuppressive medications or corticosteroids are more susceptible to fungal endocarditis. Patients on immunoglobulins, those who need in and out catheterization, and children are not at increased risk for nosocomial infective endocarditis.

A 17-year-old boy is being treated in the ICU after going into cardiac arrest during a football practice. Diagnostic testing reveals cardiomyopathy as the cause of the arrest. What type of cardiomyopathy is particularly common among young people who appear otherwise healthy? A) Dilated cardiomyopathy (DCM). B) Arrhythmogenic right ventricular cardiomyopathy (ARVC) C) Hypertrophic cardiomyopathy (HCM) D) Restrictive or constrictive cardiomyopathy (RCM)

Ans: Hypertrophic cardiomyopathy (HCM) Feedback: With HCM, cardiac arrest (i.e., sudden cardiac death) may be the initial manifestation in young people, including athletes. DCM, ARVC, and RCM are not typically present in younger adults who appear otherwise healthy.

The nurse is creating a plan of care for a patient with a cardiomyopathy. What priority goal should underlie most of the assessments and interventions that are selected for this patient? A) Absence of complications B) Adherence to the self-care program C) Improved cardiac output D) Increased activity tolerance

Ans: Improved cardiac output Feedback: The priority nursing diagnosis of a patient with cardiomyopathy would include improved or maintained cardiac output. Regardless of the category and cause, cardiomyopathy may lead to severe heart failure, lethal dysrhythmias, and death. The pathophysiology of all cardiomyopathies is a series of progressive events that culminate in impaired cardiac output. Absence of complications, adherence to the self-care program, and increased activity tolerance should be included in the care plan, but they do not have the priority of improved cardiac output.

A patient with hypertrophic cardiomyopathy (HCM) has been admitted to the medical unit. During the nurse's admission interview, the patient states that she takes over-the-counter "water pills" on a regular basis. How should the nurse best respond to the fact that the patient has been taking diuretics? A) Encourage the patient to drink at least 2 liters of fluid daily. B) Increase the patient's oral sodium intake. C) Inform the care provider because diuretics are contraindicated. D) Ensure that the patient's fluid balance is monitored vigilantly.

Ans: Inform the care provider because diuretics are contraindicated. Feedback: Diuretics are contraindicated in patients with HCM, so the primary care provider should be made aware. Adjusting the patient's sodium or fluid intake or fluid monitoring does not address this important contraindication.

.The staff educator is presenting a workshop on valvular disorders. When discussing the pathophysiology of aortic regurgitation the educator points out the need to emphasize that aortic regurgitation causes what? A) Cardiac tamponade B) Left ventricular hypertrophy C) Right-sided heart failure D) Ventricular insufficiency

Ans: Left ventricular hypertrophy Feedback: Aortic regurgitation eventually causes left ventricular hypertrophy. In aortic regurgitation, blood from the aorta returns to the left ventricle during diastole in addition to the blood normally delivered by the left atrium. The left ventricle dilates, trying to accommodate the increased volume of blood. Aortic regurgitation does not cause cardiac tamponade, right-sided heart failure, or ventricular insufficiency.

The nurse is caring for a patient who is scheduled to undergo mechanical valve replacement. Patient education should include which of the following? A) Use of patient-controlled analgesia B) Long-term anticoagulant therapy C) Steroid therapy D) Use of IV diuretics

Ans: Long-term anticoagulant therapy Feedback: Mechanical valves necessitate long-term use of required anticoagulants. Diuretics and steroids are not indicated and patient-controlled analgesia may or may be not be used in the immediate postoperative period.

A patient newly admitted to the telemetry unit is experiencing progressive fatigue, hemoptysis, and dyspnea. Diagnostic testing has revealed that these signs and symptoms are attributable to pulmonary venous hypertension. What valvular disorder should the nurse anticipate being diagnosed in this patient? A) Aortic regurgitation B) Mitral stenosis C) Mitral valve prolapse D) Aortic stenosis

Ans: Mitral stenosis Feedback: The first symptom of mitral stenosis is often dyspnea on exertion as a result of pulmonary venous hypertension. Symptoms usually develop after the valve opening is reduced by one-third to one-half its usual size. Patients are likely to show progressive fatigue as a result of low cardiac output. The enlarged left atrium may create pressure on the left bronchial tree, resulting in a dry cough or wheezing. Patients may expectorate blood (i.e., hemoptysis) or experience palpitations, orthopnea, paroxysmal nocturnal dyspnea (PND), and repeated respiratory infections. Pulmonary venous hypertension is not typically caused by aortic regurgitation, mitral valve prolapse, or aortic stenosis.

The nurse is caring for a patient with right ventricular hypertrophy and consequently decreased right ventricular function. What valvular disorder may have contributed to this patient's diagnosis? A) Mitral valve regurgitation B) Aortic stenosis C) Aortic regurgitation D) Mitral valve stenosis

Ans: Mitral valve stenosis Feedback: Because no valve protects the pulmonary veins from the backward flow of blood from the atrium, the pulmonary circulation becomes congested. As a result, the right ventricle must contract against an abnormally high pulmonary arterial pressure and is subjected to excessive strain. Eventually, the right ventricle fails. None of the other listed valvular disorders has this pathophysiological effect.

A patient has been admitted with an aortic valve stenosis and has been scheduled for a balloon valvuloplasty in the cardiac catheterization lab later today. During the admission assessment, the patient tells the nurse he has thoracolumbar scoliosis and is concerned about lying down for any extended period of time. What is a priority action for the nurse? A) Arrange for an alternative bed. B) Measure the degree of the curvature. C) Notify the surgeon immediately. D) Note the scoliosis on the intake assessment.

Ans: Notify the surgeon immediately. Feedback: Most often used for mitral and aortic valve stenosis, balloon valvuloplasty is contraindicated for patients with left atrial or ventricular thrombus, severe aortic root dilation, significant mitral valve regurgitation, thoracolumbar scoliosis, rotation of the great vessels, and other cardiac conditions that require open heart surgery. Therefore notifying the physician would be the priority over further physical assessment. An alternative bed would be unnecessary and documentation is not a sufficient response.

The nurse is preparing a patient for cardiac surgery. During the procedure, the patient's heart will be removed and a donor heart implanted at the vena cava and pulmonary veins. What procedure will this patient undergo? A) Orthotopic transplant B) Xenograft C) Heterotropic transplant D) Homograft

Ans: Orthotopic transplant Feedback: Orthotopic transplantation is the most common surgical procedure for cardiac transplantation. The recipient's heart is removed, and the donor heart is implanted at the vena cava and pulmonary veins. Some surgeons still prefer to remove the recipient's heart, leaving a portion of the recipient's atria (with the vena cava and pulmonary veins) in place. Homografts, or allografts (i.e., human valves), are obtained from cadaver tissue donations and are used for aortic and pulmonic valve replacement. Xenografts and heterotropic transplantation are not terms used to describe heart transplantation.

A cardiac surgery patient's new onset of signs and symptoms is suggestive of cardiac tamponade. As a member of the interdisciplinary team, what is the nurse's most appropriate action? A) Prepare to assist with pericardiocentesis. B) Reposition the patient into a prone position. C) Administer a dose of metoprolol. D) Administer a bolus of normal saline.

Ans: Prepare to assist with pericardiocentesis. Feedback: Cardiac tamponade requires immediate pericardiocentesis. Beta-blockers and fluid boluses will not relieve the pressure on the heart and prone positioning would likely exacerbate symptoms.

A community health nurse is presenting an educational event and is addressing several health problems, including rheumatic heart disease. What should the nurse describe as the most effective way to prevent rheumatic heart disease? A) Recognizing and promptly treating streptococcal infections B) Prophylactic use of calcium channel blockers in high-risk populations C) Adhering closely to the recommended child immunization schedule D) Smoking cessation

Ans: Recognizing and promptly treating streptococcal infections Feedback: Group A streptococcus can cause rheumatic heart fever, resulting in rheumatic endocarditis. Being aware of signs and symptoms of streptococcal infections, identifying them quickly, and treating them promptly, are the best preventative techniques for rheumatic endocarditis. Smoking cessation, immunizations, and calcium channel blockers will not prevent rheumatic heart disease.

The nurse is caring for a patient with mitral stenosis who is scheduled for a balloon valvuloplasty. The patient tells the nurse that he is unsure why the surgeon did not opt to replace his damaged valve rather than repairing it. What is an advantage of valvuloplasty that the nurse should cite? A) The procedure can be performed on an outpatient basis in a physician's office. B) Repaired valves tend to function longer than replaced valves. C) The procedure is not associated with a risk for infection. D) Lower doses of antirejection drugs are required than with valve replacement.

Ans: Repaired valves tend to function longer than replaced valves. Feedback: In general, valves that undergo valvuloplasty function longer than prosthetic valve replacements and patients do not require continuous anticoagulation. Valvuloplasty carries a risk of infection, like all surgical procedures, and it is not performed in a physician's office. Antirejection drugs are unnecessary because foreign tissue is not introduced.

The nurse is caring for a recent immigrant who has been diagnosed with mitral valve regurgitation. The nurse should know that in developing countries the most common cause of mitral valve regurgitation is what? A) A decrease in gamma globulins B) An insect bite C) Rheumatic heart disease and its sequelae D) Sepsis and its sequelae

Ans: Rheumatic heart disease and its sequelae Feedback: The most common cause of mitral valve regurgitation in developing countries is rheumatic heart disease and its sequelae.

A patient has undergone a successful heart transplant and has been discharged home with a medication regimen that includes cyclosporine and tacrolimus. In light of this patient's medication regimen, what nursing diagnosis should be prioritized? A) Risk for injury B) Risk for infection C) Risk for peripheral neurovascular dysfunction D) Risk for unstable blood glucose

Ans: Risk for infection Feedback: Immunosuppressants decrease the body's ability to resist infections, and a satisfactory balance must be achieved between suppressing rejection and avoiding infection. These drugs do not create a heightened risk of injury, neurovascular dysfunction, or unstable blood glucose levels.

A patient is admitted to the critical care unit (CCU) with a diagnosis of cardiomyopathy. When reviewing the patient's most recent laboratory results, the nurse should prioritize assessment of which of the following? A) Sodium B) AST, ALT, and bilirubin C) White blood cell differential D) BUN

Ans: Sodium Feedback: Sodium is the major electrolyte involved with cardiomyopathy. Cardiomyopathy often leads to heart failure which develops, in part, from fluid overload. Fluid overload is often associated with elevated sodium levels. Consequently, sodium levels are followed more closely than other important laboratory values, including BUN, leukocytes, and liver function tests.

A nurse is planning discharge health education for a patient who will soon undergo placement of a mechanical valve prosthesis. What aspect of health education should the nurse prioritize in anticipation of discharge? A) The need for long-term antibiotics B) The need for 7 to 10 days of bed rest C) Strategies for preventing atherosclerosis D) Strategies for infection prevention

Ans: Strategies for infection prevention Feedback: Patients with a mechanical valve prosthesis (including annuloplasty rings and other prosthetic materials used in valvuloplasty) require education to prevent infective endocarditis. Despite these infections risks, antibiotics are not used long term. Activity management is important, but extended bed rest is unnecessary. Valve replacement does not create a heightened risk for atherosclerosis.

.A patient who has undergone valve replacement surgery is being prepared for discharge home. Because the patient will be discharged with a prescription for warfarin (Coumadin), the nurse should educate the patient about which of the following? A) The need for regularly scheduled testing of the patient's International Normalized Ratio (INR) B) The need to learn to sleep in a semi-Fowler's position for the first 6 to 8 weeks to prevent emboli C) The need to avoid foods that contain vitamin K D) The need to take enteric-coated ASA on a daily basis

Ans: The need for regularly scheduled testing of the patient's International Normalized Ratio (INR) Feedback: Patients who take warfarin (Coumadin) after valve replacement have individualized target INRs; usually between 2 and 3.5 for mitral valve replacement and 1.8 and 2.2 for aortic valve replacement. Natural sources of vitamin K do not normally need to be avoided and ASA is not indicated. Sleeping upright is unnecessary.

The nurse is caring for a patient with acute pericarditis. What nursing management should be instituted to minimize complications? A) The nurse keeps the patient isolated to prevent nosocomial infections. B) The nurse encourages coughing and deep breathing. C) The nurse helps the patient with activities until the pain and fever subside. D) The nurse encourages increased fluid intake until the infection resolves.

Ans: The nurse helps the patient with activities until the pain and fever subside. Feedback: To minimize complications, the nurse helps the patient with activity restrictions until the pain and fever subside. As the patient's condition improves, the nurse encourages gradual increases of activity. Actions to minimize complications of acute pericarditis do not include keeping the patient isolated. Due to pain, coughing and deep breathing are not normally encouraged. An increase in fluid intake is not always necessary.

A patient with mitral valve prolapse is admitted for a scheduled bronchoscopy to investigate recent hemoptysis. The physician has ordered gentamicin to be taken before the procedure. What is the rationale for this? A) To prevent bacterial endocarditis B) To prevent hospital-acquired pneumonia C) To minimize the need for antibiotic use during the procedure D) To decrease the need for surgical asepsis

Ans: To prevent bacterial endocarditis Feedback: Antibiotic prophylaxis is recommended for high-risk patients immediately before and sometimes after the following invasive procedures, such as bronchoscopy. Gentamicin would not be given to prevent pneumonia, to avoid antibiotic use during the procedure, or to decrease the need for surgical asepsis.

The nurse is teaching a patient diagnosed with aortic stenosis appropriate strategies for attempting to relieve the symptom of angina without drugs. What should the nurse teach the patient? A) To eat a small meal before taking nitroglycerin B) To drink a glass of milk before taking nitroglycerin C) To engage in 15 minutes of light exercise before taking nitroglycerin D) To rest and relax before taking nitroglycerin

Ans: To rest and relax before taking nitroglycerin Feedback: The venous dilation that results from nitroglycerin decreases blood return to the heart, thus decreasing cardiac output and increasing the risk of syncope and decreased coronary artery blood flow. The nurse teaches the patient about the importance of attempting to relieve the symptoms of angina with rest and relaxation before taking nitroglycerin and to anticipate the potential adverse effects. Exercising, eating, and drinking are not recommended prior to using nitroglycerin.

To assess the dorsalis pedis artery, the nurse would use the tips of three fingers and apply light pressure to the:

Anterior surface of the foot near the ankle joint.

The nurse is caring for a client on a monitored telemetry unit. During morning assessment, the nurse notes abnormal ECG waves on the telemetry monitor. Which action would the nurse do first?

Assess the client.

Homocysteine levels have been linked to levels of?

Atherosclerosis from damage to endothelial lining of arteries

A client has undergone cardiac catheterization and will be discharged today. What information should the nurse emphasize during discharge teaching?

Avoid heavy lifting for the next 24 hours.

How is the pulmonary artery wedge pressure measured?

Balloon is inflated at distal end of pulmonary artery catheter

Vasodilation or vasoconstriction produced by an external cause will interfere with a nurse's accurate assessment of a client with peripheral vascular disease (PVD). Therefore, the nurse should: a) match the room temperature to the client's body temperature. b) maintain room temperature at 78° F (25.6° C). c) keep the client warm. d) keep the client uncovered.

C) keep the client warm. Explanation: The nurse should keep the client covered and expose only the portion of the client's body that she's assessing. The nurse should also keep the client warm by maintaining his room temperature between 68° F and 74° F (20° and 23.3° C). Extreme temperatures aren't good for clients with PVD. The valves in their arteries and veins are already insufficient, and exposing them to vast changes in temperature could affect assessment findings. Keeping the client uncovered would cause him to become chilled. Matching the room temperature to the client's body temperature is inappropriate.

What is the normal CO and normal CI?

CO = 4-8 L/min CI = 2.8-4.2 L/min

What is the difference between Cardiac Output and Cardiac Index?

CO = HR x SV CI = CO / BSA (Body Surface Area)

The term for a diagnostic test that involves injection of a contrast media into the venous system through a dorsal vein in the foot is which of the following?

Contrast phlebography

Which of the following is a diagnostic test that involves injection of a contrast media into the venous system through a dorsal vein in the foot?

Contrast phlebography

The clinic nurse caring for a client with a cardiovascular disorder is performing an assessment of the client's pulse. Which of the following steps is involved in determining the pulse deficit?

Count the heart rate at the apex.

The nurse is caring for an elderly client with left-sided heart failure. When auscultating lung sounds, which adventitious sound is expected?

Crackles

What are the cardiac biomarkers that are released when myocardial cells become necrotic from ischemia or injury/trauma?

Creatinine Kinase (CK), Creatinine Kinase Isoenzymes (CK-MB), Myoglobin, Tropinin T, Tropinin I

What overall factor can cause decrease in CVP?

Decrease in blood volume (hypovolemia, shock, dehydration, etc.)

The most important reason for a nurse to encourage a client with peripheral vascular disease to initiate a walking program is that this form of exercise:

Decreases venous congestion

What is the purpose of an electrophysiology study?

Determine source of arrhythmias

How is a chest x-ray used as a diagnostic tool?

Determines size, contour, and position of heart

Why is NS used instead of dextrose?

Dextrose can grow bacteria!

The nurse performing an assessment on a patient who has arterial insufficiency of the legs and an ulcer on the left great toe would expect to find which of the following characteristics?

Diminished or absent pulses

Which end of a pulmonary artery catheter is inserted into the pulmonary artery?

Distal End (Usually Yellow)

What is the difference between an echocardiogram and a trans-esophageal echocardiogram?

Echocardiogram - Non-invasive procedure where gel is placed on chest and a transducer is moved across gel on chest Trans-Esophageal Echocardiogram - Invasive procedure where scope is inserted into esophagus

A client with venous insufficiency asks the nurse what they can do to decrease their risk of complications. What advice should the nurse provide to clients with venous insufficiency?

Elevate the legs periodically for at least 15 to 20 minutes.

What is cardiomegaly?

Enlarged Heart

You are monitoring the results of laboratory tests performed on a client admitted to the cardiac ICU with a diagnosis of myocardial infarction. Which test would you expect to show elevated levels?

Enzymes

Which of the following statements is accurate regarding Reynaud's disease?

Episodes may be triggered by unusual sensitivity to cold

What is the purpose of cardiac stress testing?

Evaluate myocardial blood flow and perfusion

What is the purpose of a calcium coronary storage test?

Evaluates the buildup of calcium in plaque on the walls of the coronary arteries. Calcium is normally not found in walls of corner arteries and is an indicator of coronary artery disease!

A nurse is instructing a client about using antiembolism stockings. Antiembolism stockings help prevent deep vein thrombosis (DVT) by:

Forcing blood into the deep venous system.

The nurse is completing a cardiac assessment. Upon auscultation, the nurse hears a grating sound using the diaphragm of the stethoscope. How will the nurse best document this finding?

Friction rub

Which molecule transports cholesterol away from the tissues and cells of the arterial wall to the liver for excretion?

HDL

You are doing an admission assessment on a client who is having outpatient testing done for cardiac problems. What should you ask this client during your assessment?

Have you had any episodes of dizziness or fainting?"

What is the gold standard test for coronary artery disease?

Heart Catheterization (Cath Lab)

The nurse instructor is teaching a group of nursing students about adventitious heart sounds. The instructor explains that auscultation of the heart requires familiarization with normal and abnormal heart sounds. What would the instructor tell these students a ventricular gallop indicates in an adult?

Heart failure

The nurse is reviewing the results of the patient's echocardiogram and observes that the ejection fraction is 35%. The nurse anticipates that the patient will receive treatment for what condition?

Heart failure

Which diagnostic test is more specific for cardiac function? C-reactive Protein or High Sensitivity C-reactive Protein?

High Sensitivity C-Reactive Protein

A client who underwent total hip replacement exhibits a red, painful area on the calf of the affected leg. What test validates presence of thromboembolism?

Homans'

The nurse is caring for a patient who has started anticoagulant therapy with warfarin (Coumadin). When does the nurse understand that therapeutic benefits will begin?

In 3 to 5 days

What overall factor can cause increase in CVP?

Increase in blood volume (fluid overload, right side heat failure, vasoconstriction)

Why is aspirin and NSAIDS important when identifying routine meds prior to heart catheterization?

Increase risk of bleeding

A nurse is developing a nursing care plan for a client with peripheral arterial disease. Which of the following will be the priority nursing diagnosis?

Ineffective peripheral tissue perfusion

A client is diagnosed with deep vein thrombosis (DVT). Which nursing diagnosis should receive highest priority at this time?

Ineffective peripheral tissue perfusion related to venous congestion

Higher levels of C-reactive protein indicate?

Inflammation (Produced in liver in response to inflammation)

A nurse is teaching a patient newly diagnosed with arterial insufficiency. Which of the following terms should the nurse use to refer to leg pain that occurs when the patient is walking?

Intermittent claudication

Which of the following is the hallmark symptom for peripheral arterial disease (PAD) in the lower extremity?

Intermittent claudication

Which of the following terms refers to a muscular, cramplike pain in the extremities consistently reproduced with the same degree of exercise and relieved by rest?

Intermittent claudication

Which of the following statements about myocardial infarction pain is incorrect?

It is relieved by rest and inactivity

The nurse is reviewing the morning laboratory test results for a client with cardiac problems. Which finding is a priority to report to the health care provider?

K+ 3.1 mEq/L

The nurse is caring for a client who is scheduled for a transesophageal echocardiogram. What nursing intervention is a priority after the procedure?

Keep the head of the bed elevated 45 degrees and keep NPO until return of the gag reflex.

What cardiac function is mainly evaluated with echocardiogram?

Left Ventricular Function (Wall motion, ejection fraction)

What does the distal port of PA catheter with balloon inflated measure? Where is this catheter placed?

Left Ventricular Preload - Located in Pulmonary Artery

After a physical examination, the provider diagnosed a patient with a grade 4 heart murmur. When ausculatating a murmur, what does the nurse expect to hear?

Loud and may be associated with a thrill sound similar to (a purring cat).

The nurse is administering a beta blocker to a patient in order to decrease automaticity. Which medication will the nurse administer?

Metoprolol

Should the balloon on the pulmonary artery catheter be left inflated?

NO!

What is Brain Natriuretic Peptide and why is it released?

Neurohormone released by ventricles in response to increase preload (stretch) with resulting elevated ventricular pressure

A nurse is assessing a client's right lower leg, which is wrapped with an elastic bandage. Which signs and symptoms suggest circulatory impairment?

Numbness, cool skin temperature, and pallor

Which of the following are risk factors for venous disorders of the lower extremities?

Obesity

The nurse cares for a client with clubbing of the fingers and toes. The nurse should complete which action given these findings?

Obtain an oxygen saturation level.

A nurse is caring for a client following an arterial vascular bypass graft in the leg. Over the next 24 hours, what should the nurse plan to assess?

Peripheral pulses every 15 minutes following surgery

The transducers must be level to which anatomical location for precise measurement?

Phlebostatic Axis (4 ICS - Mid Axillary Line)

What does the Central Venous Pressure (CVP) measure? Where is the catheter placed?

Preload on right side of heart - Placed in right atrium

If a patent is scheduled for a cardiac stress test and complains of chest pain prior to the procedure, should the nurse allow the patient to continue the stress test or prevent the patient from performing the stress test?

Prevent the patient from performing the stress test

Why do we continuously flush the catheter with IV normal saline?

Prevent the tube from becoming clogged

Which end of a pulmonary artery catheter is placed in right atrium?

Proximal (Usually Blue)

The nurse is assessing a client taking an anticoagulant. What nursing intervention is most appropriate for a client at risk for injury related to side effects of medication enoxaparin?

Report any incident of bloody urine, stools, or both.

What does the distal port of PA catheter with balloon deflated measure? Where is the catheter placed?

Right Ventricular Afterload- Located in Pulmonary Artery

Which of the following is the most effective intervention for preventing progression of vascular disease?

Risk factor modification

A nurse is providing education about the prevention of arterial constriction to a client with peripheral arterial disease. Which of the following includes priority information the nurse would give to the client?

Stop smoking.

What should the nurse do to manage the persistent swelling in a patient with severe lymphangitis and lymphadenitis?

Teach the patient how to apply a graduated compression stocking.

You are evaluating the expected outcomes on a client who is recovering from a cardiac catheterization. What is an expected outcome that you would evaluate?

The client and family understands the discharge instructions.

A pregnant client who developed deep vein thrombosis (DVT) in her right leg is receiving heparin I.V. on the medical floor. Physical therapy is ordered to maintain her mobility and prevent additional DVT. A nursing assistant working on the medical unit helps the client with bathing, range-of-motion exercises, and personal care. Which collaborative multidisciplinary considerations should the care plan address?

The client is at risk for heparin-induced thrombocytopenia; therefore, the care plan should include reporting evidence of bleeding or easy bruising.

The client is admitted for a scheduled cardiac catheterization. On the morning of the procedure, while assessing the client's morning laboratory values, the nurse notes a blood urea nitrogen (BUN) of 34 mg/dL and a creatinine of 4.2 mg/dL. What priority reason will the nurse notify the healthcare provider?

The client is at risk for renal failure due to the contrast agent that will be given during the procedure.

What part of the pulmonary artery catheter measures core body temperature?

Thermistor

A nurse is caring for a dying client following myocardial infarction. The client is experiencing apnea with a falling blood pressure of 60 per palpation. Which documentation of pulse quality does the nurse anticipate?

Thready pulse

Which type of echocardiogram produces a better quality image of the heart?

Trans-Esophageal Echocardiogram

T/F - We want a high level of HDL?

True!

The nurse is caring for a patient with venous insufficiency. What should the nurse assess the patient's lower extremities for?

Ulceration

What are the symptoms a nurse should assess for in a patient with lymphedema as a result of impaired nutrition to the tissue?

Ulcers and infection in the edematous area

How does an echocardiogram work?

Uses sound waves to produce an image of the heart

Why is Viagra important when identifying routine meds prior to heart catheterization?

Vasodilator that doesn't allow body to respond to increase BP - Therefore, not good when given with other vasodilators (Ex. Nitroglycerin)

The nurse teaches the patient with peripheral vascular disease (PVD) to refrain from smoking because nicotine causes which of the following?

Vasospasm

Describe the tubing of a hemodynamic monitoring catheter?

Very stiff

A nurse is teaching a client who is having a valuloplasty tomorrow. The client asks what the advantage is for having a tissue valve replacement instead of a mechanical valve. The correct answer by the nurse is which of the following? a) "A tissue valve is less likely to generate blood clots, and so long-term anticoagulation therapy is not required." b) "A tissue valve does not become infected as easily as mechanical valves." c) "A tissue valve is thought to be more durable and so requires replacement less often." d) "A tissue valve does not deteriorate as easily as mechanical valves."

a) "A tissue valve is less likely to generate blood clots, and so long-term anticoagulation therapy is not required." Tissue valves are less likely to generate thromoemboli, so long-term anticoagulation is not required. Mechanical valves do not deteriorate or become infected as easily as tissue valves. They are thought to be more durable than tissue valves and so require replacement less often.

Your client is being prepared for echocardiography when he asks you why he needs to have this test. What would be your best response? a) "Echocardiography is a way of determining the functioning of the left ventricle of your heart." b) "This test can tell us a lot about your heart." c) "This test will find any congenital heart defects." d) "Echocardiography will tell your doctor if you have cancer of the heart."

a) "Echocardiography is a way of determining the functioning of the left ventricle of your heart." Echocardiography uses ultrasound waves to determine the functioning of the left ventricle and to detect cardiac tumors, congenital defects, and changes in the tissue layers of the heart. All answers are correct. Option C is the best answer because it addresses the client's question without making him anxious or minimizing the question.

A nurse is reviewing self-care measures for a client with peripheral vascular disease. Which statement indicates proper self-care measures? a) "I have my wife look at the soles of my feet each day." b) "I like to soak my feet in the hot tub every day." c) "I stopped smoking and use only chewing tobacco." d) "I walk only to the mailbox in my bare feet."

a) "I have my wife look at the soles of my feet each day." Explanation: A client with peripheral vascular disease should examine his feet daily for redness, dryness, or cuts. If a client isn't able to do this examination on his own, then a caregiver or family member should help him. A client with peripheral vascular disease should avoid hot tubs because decreased sensation in the feet may make him unable to tell if the water is too hot. The client should always wear shoes or slippers on his feet when he is out of bed to help minimize trauma to the feet. Any type of nicotine, whether it's from cigarettes or smokeless tobacco, can cause vasoconstriction and further decrease blood supply to the extremities.

The nurse in a cardiac clinic is taking vital signs of a 58-year-old man who is 3 months status post myocardial infarction (MI). While the physician is seeing the client, the client's spouse approaches the nurse and asks about sexual activity. "We are too afraid he will have another heart attack, so we just don't have sex anymore." The nurse's best response is which of the following? a) "The physiologic demands are greatest during orgasm and are equivalent to walking 3 to 4 miles per hour on a treadmill." b) "The medications will prevent your husband from having an erection." c) "It is usually better to just give up sex after a heart attack." d) "Having an orgasm is very strenuous and your husband must be in excellent physical shape before attempting it."

a) "The physiologic demands are greatest during orgasm and are equivalent to walking 3 to 4 miles per hour on a treadmill." The physiologic demands are greatest during orgasm. The level of activity is equivalent to walking 3 to 4 miles per hour on a treadmill. Erectile dysfunction may be a side effect of beta-blockers, but other medications may be substituted.

Beginning warfarin concomitantly with heparin can provide a stable INR by which day of heparin treatment? a) 5 b) 2 c) 3 d) 4

a) 5 Explanation: Beginning warfarin concomitantly with heparin can provide a stable INR by day 5 of heparin treatment, at which time the heparin maybe discontinued.

The client teaching instructions for a 57-year-old male client with thrombophlebitis who is being discharged should include which of the following? Select all that apply. a) Ambulate as tolerated. b) Avoid elevating affected extremity. c) Take anticoagulant therapy when symptoms occur. d) Avoid sitting for too long. e) Perform leg exercises each hour.

a) Ambulate as tolerated. d) Avoid sitting for too long. e) Perform leg exercises each hour. Nurses instruct clients with thrombophlebitis to prevent recurrences by being active, avoiding knee bending or leg crossing, elevating legs periodically, and taking long-term anticoagulant therapy exactly as prescribed. Clients should also watch for and report signs that indicate impaired clotting: nosebleeds, bleeding gums, rectal bleeding, easy bruising, and prolonged oozing from minor cuts.

A nurse is caring for a client who is exhibiting signs and symptoms characteristic of a myocardial infarction (MI). Which statement describes priorities the nurse should establish while performing the physical assessment? a) Assess the client's level of pain and administer prescribed analgesics. b) Ensure that the client's family is kept informed of his status. c) Prepare the client for pulmonary artery catheterization. d) Assess the client's level of anxiety and provide emotional support.

a) Assess the client's level of pain and administer prescribed analgesics. Explanation: The cardinal symptom of MI is persistent, crushing substernal pain or pressure. The nurse should first assess the client's pain and prepare to administer nitroglycerin or morphine for pain control. The client must be medically stabilized before pulmonary artery catheterization can be used as a diagnostic procedure. Anxiety and a feeling of impending doom are characteristic of MI, but the priority is to stabilize the client medically. Although the client and his family should be kept informed at every step of the recovery process, this action isn't the priority when treating a client with a suspected MI.

A patient has undergone a cardiac catheterization. He is to be discharged today. What information should the nurse emphasize during discharge teaching? a) Avoid heavy lifting for the next 24 hours. b) Take a tub bath, rather than a shower. c) Bend only at the waist. d) New bruising at the puncture site is normal.

a) Avoid heavy lifting for the next 24 hours. For the next 24 hours, the patient should not bend at the waist, strain, or life heavy objects. The patient should avoid tub baths, but shower as desired. The patient should call her the health care provider if she has any bleeding, swelling, new bruising, or pain from her procedure puncture site, or a temperature of 101.5 degrees Fahrenheit or more.

A patient is admitted with aortic regurgitation. Which of the following medication classifications are contraindicated since they can cause bradycardia and decrease ventricular contractility? a) Calcium channel blockers b) Beta blockers c) Ace inhibitors d) Nitrates

a) Calcium channel blockers The calcium channel blockers diltiazem (Cardizem) and verapamil (Calan, Isoptin) are contraindicated for patients with aortic regurgitation as they decrease ventricular contractility and may cause bradycardia.

The nurse accompanies a client to an exercise stress test. The client can achieve the "target heart rate," but the ECG leads show an ST-segment elevation. The nurse recognizes this as a "positive" stress test, and will begin to prepare the client for which of the following procedures? a) Cardiac catheterization b) Transesophageal echocardiogram c) Pharmacologic stress test d) Telemetry monitoring

a) Cardiac catheterization An elevated ST-segment means an evolving myocardial infarction. A cardiac catheterization would be the logical next step.

A 62-year-old female who is 2 weeks CABG returns to her cardiologist due to new symptoms, including heaviness in her chest and pain between her breasts. She reports that leaning forward decreases the pain. The cardiologist admits her to the hospital to rule out pericarditis. Which of the following is a contributing cause to pericarditis? Select all that apply. a) Chest trauma b) Cardiac surgery c) Tuberculosis d) Myocarditis e) Pneumonia

a) Chest trauma b) Cardiac surgery c) Tuberculosis d) Myocarditis Pericarditis usually is secondary to endocarditis, myocarditis, chest trauma, or MI (heart attack) or develops after cardiac surgery.

The nurse is performing a neurovascular assessment of a client's injured extremity. Which of the following would the nurse report? a) Dusky or mottled skin color b) Positive distal pulses c) Skin warm to touch d) Capillary refill of 3 seconds

a) Dusky or mottled skin color Normally, skin color would be similar to the color in other body areas. Pale or dusky skin color indicates an abnormality that needs to be reported. Presence of pulses, capillary refill of 3 seconds, and warm skin are normal findings.

The nurse is assessing the client newly prescribed Lasix 20mg daily for 3+ pitting edema. To evaluate the effectiveness of diuretic therapy, which of the following would be documented? a) Edema b) Blood pressure c) Urine output d) Weight

a) Edema The best method to evaluate the effectiveness of diuretic therapy is to note a decrease in edema. Weight, blood pressure, and urine output all are affected by diuretic therapy, but the therapeutic goal is to decrease the edema.

The nurse is caring for a client diagnosed with infective endocarditis and awaiting blood culture results. The client asks, "Where did I pick up these bacteria?"The nurse is most safe to speculate which of the following? a) From a break in the skin b) From ingestion of a food c) From droplets from a cough d) From the fecal-oral route

a) From a break in the skin The microorganisms that cause infective endocarditis include bacteria and fungi. Streptococci and staphylococci are the bacteria most frequently responsible for this disorder. Both bacteria are abundantly found on the skin. These organisms are not found in the other locations.

Identify which of the following as an age-related change associated with conduction system of the heart? a) Heart block b) Murmur c) Thrills d) Tachycardia

a) Heart block Age-related changes to the conduction system may include bradycardia and heart block. Age-related changes to the heart valves include the presence of a murmur or thrill.

The nurse suspects a diagnosis of mitral valve regurgitation when what type of murmur is heard on auscultation? a) High-pitched blowing sound at the apex b) Diastolic murmur at the left sternal border c) Mitral click d) Low-pitched diastolic murmur at the apex

a) High-pitched blowing sound at the apex Mitral valve regurgitation is associated with a systolic murmur, which is a high-pitched, blowing sound at the apex.

A client who underwent total hip replacement exhibits a red, painful area on the calf of the affected leg. What test validates presence of thromboembolism? a) Homans' b) Rinne c) Romberg's d) Phalen's

a) Homans' Explanation: A positive Homans' sign, or pain in the calf elicited upon flexion of the ankle with the leg straight, indicates the presence of a thrombus. Testing for Romberg's sign assesses cerebellar function. Phalen's test assesses carpal tunnel syndrome. The Rinne test compares air and bone conduction in both ears to screen for or confirm hearing loss.

The nurse is interviewing a client who is complaining of chest pain. Which of the following questions related to the client's history are most important to ask? Select all that apply. a) How would you describe your symptoms? b) Do you have any children? c) How did your mother die? d) Are you allergic to any medications or foods?

a) How would you describe your symptoms? c) How did your mother die? d) Are you allergic to any medications or foods? During initial assessment, the nurse should obtain important information about the client's history that focuses on a description of the symptoms before and during admission, family medical history, prescription and nonprescription drug use, and drug and food allergies.

The nurse is caring for a patient in the ICU who is being monitored with a central venous pressure (CVP) catheter. The nurse records the patient's CVP as 8 mm Hg. The nurse understands that this finding indicates the patient is experiencing which of the following? a) Hypervolemia b) Excessive blood loss c) Overdiuresis d) Left-sided heart failure (HF)

a) Hypervolemia The normal CVP is 2 to 6 mm Hg. A CVP greater than 6 mm Hg indicates an elevated right ventricular preload. Many problems can cause an elevated CVP, but the most common is hypervolemia (excessive fluid circulating in the body) or right-sided HF. In contrast, a low CVP (<2 mm Hg) indicates reduced right ventricular preload, which is most often from hypovolemia.

Age-related changes associated with the cardiac system include which of the following? Select all that apply. a) Increased size of the left atrium b) Myocardial thinning c) Endocardial fibrosis d) Increase in the number of SA node cells

a) Increased size of the left atrium c) Endocardial fibrosis Age-related changes associated with the cardiac system include endocardial fibrosis, increased size of the left atrium, decreased number of SA node cells, and myocardial thickening.

The nurse is caring for a client on the cardiac unit. Which change of condition may indicate potential increasing of right-side heart failure? Select all that apply. a) Increased weakness on ambulation b) Jugular vein distention c) Edema changed from a 3+ to a 1+ d) One-pound weight loss e) Increased palpitations f) Increased dyspnea

a) Increased weakness on ambulation b) Jugular vein distention e) Increased palpitations f) Increased dyspnea A change in assessment finding may indicate an increase in heart failure. Right-sided heart failure symptoms include jugular vein distention, increased dyspnea, increased palpitations, and an increased weakness on ambulation. Edema is a common sign of right-sided heart failure, but changing from a 3+ to 1+ is improvement in condition. Weight loss is also improvement in condition.

A nurse is performing a cardiac assessment on an elderly client. Which finding warrants further investigation? a) Irregularly irregular heart rate b) Increased PR interval c) Fourth heart sound (S4) d) Orthostatic hypotension

a) Irregularly irregular heart rate An irregularly irregular heart rate indicates atrial fibrillation and should be investigated further. It's normal for an elderly client to have a prolonged systole, which causes an S4 heart sound. It's also normal for an elderly client to have slowed conduction, causing an increased PR interval. As a person ages, it's normal for baroreceptors in the body to decrease their response to changes in body position, which can cause orthostatic hypotension.

A patient with a history of valvular disease has just arrived in the PACU after a percutaneous balloon valvuloplasty. Which intervention should the recovery nurse implement? a) Keep the patient's affected leg straight. b) Monitor the patient's chest drainage. c) Evaluate the patient's endotracheal lip line. d) Assess the patient's chest tube output.

a) Keep the patient's affected leg straight. Balloon valvuloplasty is performed in the cardiac catheterization laboratory. A catheter is inserted into the femoral artery. The patient must keep the affected leg straight to prevent hemorrhage at the insertion site. It is not an open heart surgery requiring chest tubes nor a chest dressing. ET tubes are placed when someone has general anesthesia, and this procedure is performed using light or moderate sedation.

When caring for a client with a diagnosis of aortic aneurysm scheduled for surgery, what would be most important for the nurse to monitor? a) Level of consciousness, characteristics of pain, and signs of hemorrhage or dissection b) Intake and output, nutrition level, respirations, and characteristics of pain c) Cultural needs, characteristics of pain, and signs of hemorrhage or dissection d) BP, pulse, respirations, and signs of hemorrhage or dissection

a) Level of consciousness, characteristics of pain, and signs of hemorrhage or dissection Explanation: The nurse monitors BP, pulse, hourly urine output, skin color, level of consciousness, and characteristics of pain for signs of hemorrhage or dissection. Assessing respirations, nutritional levels, and cultural needs are important but not the most important assessments for the nurse to make.

On auscultation, the nurse suspects a diagnosis of mitral valve stenosis when which of the following is heard? a) Low-pitched, rumbling diastolic murmur at the apex of the heart b) High-pitched blowing sound at the apex c) Mitral valve click d) Diastolic murmur at the left sternal border of the heart

a) Low-pitched, rumbling diastolic murmur at the apex of the heart The murmur is caused by turbulent blood flow through the abnormally tight valve opening. A low-pitched, rumbling, diastolic murmur (heard on S2) is heard best at the apex. A loud S1, due to abrupt closure of the mitral valve, and an early diastolic opening snap can be heard. The snap is the premature opening of the stenotic mitral valve.

Which of the following nursing interventions should a nurse perform to reduce cardiac workload in a patient diagnosed with myocarditis? a) Maintain the patient on bed rest. b) Elevate the patient's head. c) Administer a prescribed antipyretic. d) Administer supplemental oxygen.

a) Maintain the patient on bed rest. The nurse should maintain the patient on bed rest to reduce cardiac workload and promote healing. Bed rest also helps decrease myocardial damage and the complications of myocarditis. The nurse should administer supplemental oxygen to relieve tachycardia that may develop from hypoxemia. If the patient has a fever, the nurse should administer a prescribed antipyretic along with independent nursing measures such as minimizing layers of bed linen, promoting air circulation and evaporation of perspiration, and offering oral fluids. The nurse should elevate the patient's head to promote maximal breathing potential.

The nurse is evaluating the expected outcomes following thrombolytic therapy for a right leg deep vein thrombosis. Which of the following findings confirms a positive outcome?(Select all that apply.) a) No bleeding or bruising noted b) Right extremity pink c) Right extremity comparable in size to left d) Client denies pain e) Homan's sign positive f) Pedal pulse thready

a) No bleeding or bruising noted b) Right extremity pink c) Right extremity comparable in size to left d) Client denies pain Evaluation of the expected outcome of thrombolytic therapy includes restoring blood flow to the extremity. Findings include no pain from impaired circulation, a pink extremity of comparable size, and no bleeding from complications of the thrombolytic medication. A thready pulse would indicate impaired circulation, and a positive Homan's sign would indicate a continuing thrombus.

A client has been diagnosed with peripheral arterial occlusive disease. Which of the following instructions is appropriate for the nurse to give the client for promoting circulation to the extremities? a) Participate in a regular walking program. b) Massage the calf muscles if pain occurs. c) Use a heating pad to promote warmth. d) Keep the extremities elevated slightly.

a) Participate in a regular walking program. Explanation: Clients diagnosed with peripheral arterial occlusive disease should be encouraged to participate in a regular walking program to help develop collateral circulation. They should be advised to rest if pain develops and to resume activity when pain subsides. Extremities should be kept in a dependent position to promote circulation; elevation of the extremities will decrease circulation. Heating pads should not be used by anyone with impaired circulation to avoid burns. Massaging the calf muscles will not decrease pain. Intermittent claudication subsides with rest.

A nurse is preparing to assess a patient for postural BP changes. Which of the following indicates the need for further education? a) Positioning the patient supine for 10 minutes prior to taking the initial BP and HR b) Letting 30 seconds elapse after each position change before measuring BP and heart rate (HR) c) Obtaining the supine measurements prior to the sitting and standing measurements d) Taking the patient's BP with the patient sitting on the edge of the bed with feet dangling

a) Positioning the patient supine for 10 minutes prior to taking the initial BP and HR The following steps are recommended when assessing patients for postural hypotension: Position the patient supine for 10 minutes before taking the initial BP and HR measurements; reposition the patient to a sitting position with legs in the dependent position, wait 2 minutes then reassess both BP and HR measurements; if the patient is symptom free or has no significant decreases in systolic or diastolic BP, assist the patient into a standing position, obtain measurements immediately and recheck in 2 minutes; continue measurements every 2 minutes for a total of 10 minutes to rule out postural hypotension. Return the patient to supine position if postural hypotension is detected or if the patient becomes symptomatic. Document HR and BP measured in each position (e.g., supine, sitting, and standing) and any signs or symptoms that accompany the postural changes.

A nursing student is assigned to the medical intensive care unit for the first time. The nurse preceptor asks the student to listen to a water-hammer pulse. The nursing student knows that the sound will resemble which of the following? a) Quick, sharp strokes that suddenly collapse b) Low-pitched diastolic murmur at the apex c) High-pitched blowing sound at the apex d) Mitral click

a) Quick, sharp strokes that suddenly collapse With the water-hammer (Corrigan's) pulse, the pulse strikes the palpating finger with a quick, sharp stroke and then suddenly collapses.

Which of the following is accurate regarding the effects of nicotine and tobacco smoke on the body? Select all that apply. a) Reduces circulation to the extremities b) Impairs transport and cellular use of oxygen c) Causes vasospasm d) Increases blood viscosity e) Decreases blood viscosity

a) Reduces circulation to the extremities b) Impairs transport and cellular use of oxygen c) Causes vasospasm d) Increases blood viscosity Explanation: Nicotine from tobacco products causes vasospasm and can dramatically reduce circulation to the extremities. Tobacco smoke also impairs transport and cellular use of oxygen and increases blood viscosity.

The nurse is screening a patient prior to a magnetic resonance angiogram (MRA) of the heart. Which of the following actions should the nurse complete prior to the patient undergoing the procedure? Select all that apply. a) Remove the patient's jewelry. b) Offer the patient a headset to listen to music during the procedure. c) Remove the patient's Transderm Nitro patch. d) Sedate the patient prior to the procedure. e) Position the patient on his/her stomach for the procedure.

a) Remove the patient's jewelry. b) Offer the patient a headset to listen to music during the procedure. c) Remove the patient's Transderm Nitro patch. Transdermal patches that contain a heat-conducting aluminized layer (e.g., NicoDerm, Androderm, Transderm Nitro, Transderm Scop, Catapres-TTS) must be removed before MRA to prevent burning of the skin. A patient who is claustrophobic may need to receive a mild sedative before undergoing an MRA. During an MRA, the patient is positioned supine on a table that is placed into an enclosed imager or tube containing the magnetic field. Patients are instructed to remove any jewelry, watches, or other metal items (e.g., ECG leads). An intermittent clanking or thumping that can be annoying is generated by the magnetic coils, so the patient may be offered a headset to listen to music.

A nurse should be prepared to manage complications following abdominal aortic aneurysm resection. Which complication is most common? a) Renal failure b) Graft occlusion c) Hemorrhage and shock d) Enteric fistula

a) Renal failure Explanation: Renal failure commonly occurs if clamping time is prolonged, cutting off the blood supply to the kidneys. Hemorrhage and shock are the most common complications before abdominal aortic aneurysm resection, and they occur if the aneurysm leaks or ruptures. Graft occlusion and enteric fistula formation are rare complications of abdominal aortic aneurysm repair.

A patient was admitted to the hospital with a diagnosis of aortic regurgitation. On assessment, the nurse notes the following positive indicators for the disease process. Select all that apply. a) The pulse has a rapid upstroke, then collapses b) Systolic pressure in the lower extremities is lower than in the upper extremities c) Visible neck vein pulsations d) The presence of a diastolic murmur e) Shortened pulse pressure

a) The pulse has a rapid upstroke, then collapses c) Visible neck vein pulsations d) The presence of a diastolic murmur Pulse pressure widens and systolic blood pressure in the lower extremities is higher than in the upper extremities as a result of progressive left ventricular failure.

A client has a blockage in the proximal portion of a coronary artery. After learning about treatment options, the client decides to undergo percutaneous transluminal coronary angioplasty (PTCA). During this procedure, the nurse expects to administer an: a) anticonvulsant. b) antihypertensive. c) anticoagulant. d) antibiotic.

a) anticoagulant. Explanation: During PTCA, the client receives heparin, an anticoagulant, as well as calcium agonists, nitrates, or both, to reduce coronary artery spasm. Nurses don't routinely give antibiotics during this procedure; however, because the procedure is invasive, the client may receive prophylactic antibiotics to reduce the risk of infection. An antihypertensive may cause hypotension, which should be avoided during the procedure. An anticonvulsant isn't indicated because this procedure doesn't increase the risk of seizures.

To check for arterial insufficiency when a client is in a supine position, the nurse should elevate the extremity at a 45-degree angle and then have the client sit up. The nurse suspects arterial insufficiency if the assessment reveals: a) dependent pallor. b) elevational rubor. c) a 30-second filling time for the veins. d) no rubor for 10 seconds after the maneuver.

a) dependent pallor. Explanation: If arterial insufficiency is present, elevation of the limb would yield a pallor from the lack of circulation. Rubor and increased venous filling time would suggest venous problems secondary to venous trapping and incompetent valves.

A patient with infective endocarditis of a prosthetic mitral valve returns to the emergency department with a second episode of left-sided weakness and visual changes. The nurse expects that collaborative management of the patient will include a) surgical valve replacement b) administration of anticoagulants c) higher than usual antibiotic dosages d) embolectomy

a) surgical valve replacement Aortic or mitral valve debridement, excision, or replacement is required in patients with more than one serious systemic embolic episode.

When caring for a client with a diagnosis of aortic aneurysm scheduled for surgery, what would be most important for the nurse to monitor? a) Level of consciousness, characteristics of pain, and signs of hemorrhage or dissection b) BP, pulse, respirations, and signs of hemorrhage or dissection c) Cultural needs, characteristics of pain, and signs of hemorrhage or dissection d) Intake and output, nutrition level, respirations, and characteristics of pain

a)Level of consciousness, characteristics of pain, and signs of hemorrhage or dissection Explanation: The nurse monitors BP, pulse, hourly urine output, skin color, level of consciousness, and characteristics of pain for signs of hemorrhage or dissection. Assessing respirations, nutritional levels, and cultural needs are important but not the most important assessments for the nurse to make.

The classic ECG changes that occur with an MI include all of the following except:

an absent P wave

An antidote for propranolol hydrochloride (a beta-adrenergic blocker) that is used to treat bradycardia is:

atropine.

A patient with aortic valve endocarditis develops dyspnea, crackles in the lungs, and restlessness. The graduate nurse discusses this development with the nurse preceptor. The preceptor is assured when the graduate nurse states: a) "I instructed the patient to do coughing and deep breathing and I will reassess in 30 minutes." b) "I anticipated this complication and I will call the doctor right now." c) "I placed the patient in a semi-Fowler's position and made him NPO." d) "I told the patient that this is a normal complication and to take deep breaths."

b) "I anticipated this complication and I will call the doctor right now." With right-sided heart endocarditis, the nurse assesses for signs and symptoms of organ damage such as stroke, meningitis, heart failure, myocardial infarction, glomerulonephritis, and splenomegaly. This requires further assessment and collaborative interventions to prevent further deterioration. The other actions are not appropriate at this time.

A client complains of leg pain brought on by walking several blocks — a symptom that first arose several weeks earlier. The client's history includes diabetes mellitus and a two-pack-per-day cigarette habit for the past 42 years. The physician diagnoses intermittent claudication and orders pentoxifylline (Trental), 400 mg three times daily with meals. Which instruction concerning long-term care should the nurse provide? a) "See the physician if complications occur." b) "Practice meticulous foot care." c) "Consider cutting down on your smoking." d) "Reduce your level of exercise."

b) "Practice meticulous foot care." Explanation: Intermittent claudication and other chronic peripheral vascular diseases reduce oxygenation to the feet, making them susceptible to injury and poor healing. Therefore, meticulous foot care is essential. The nurse should teach the client to bathe his feet in warm water and dry them thoroughly, cut the toenails straight across, wear well-fitting shoes, and avoid taking medications without the approval of a physician. Because nicotine is a vasoconstrictor, this client should stop smoking, not just consider cutting down. Daily walking is beneficial to clients with intermittent claudication. To evaluate the effectiveness of the therapeutic regimen, this client should see the physician regularly, not just when complications occur.

The nurse is aware that age-related changes in the heart muscle put the elderly at risk for dyspnea, angina, and syncope. Which of the following is an age-related change in the cardiovascular system that affects the sympathetic nervous system? a) An increased contractility response to exercise b) A decreased response to beta-blockers c) Decreased time for the heart rate to return to baseline d) Tachycardia

b) A decreased response to beta-blockers The sympathetic nervous system exhibits structural and functional changes that are age-related. Heart rate will decrease, and it will take longer for the heart rate to return to baseline. Refer to Table 12-1 in the text.

Which of the following would be a factor that may decrease myocardial contractility? a) Administration of digoxin (Lanoxin) b) Acidosis c) Sympathetic activity d) Alkalosis

b) Acidosis Contractility is depressed by hypoxemia, acidosis, and certain medications, such as beta-adrenergic blocking medications. Contractility is enhanced by sympathetic neuronal activity, and certain medications, such as Lanoxin.

Which of the following diagnostic tests are used to quantify venous reflux and calf muscle pump ejection? a) Lymphangiography b) Air plethysmography c) Lymphoscintigraphy d) Contrast phlebography

b) Air plethysmography Explanation: Air plethysmography is used to quantify venous reflux and calf muscle pump action. Contrast phlebography involves injecting a radiopaque contrast agent into the venous system. Lymphoscintigraphy is done when a radioactively labeled colloid is injected subcutaneously in the second interdigital space. The extremity is then exercised to facilitate the uptake of the colloid by the lymphatic system, and serial images are obtained at present intervals. Lymphoangiography provides a way of detecting lymph node involvement resulting from metastatic carcinoma, lymphoma, or infection in sites that are otherwise inaccessible to the examiner except by surgery.

A home health nurse is seeing an elderly female client for the first time. During the physical assessment of the client's feet, the nurse notes several circular ulcers around the tips of the toes on both feet. The bases of the ulcers are pale, and the client reports the ulcers to be very painful. From these assessment findings, the nurse suspects that the cause of the ulcers is which of the following? a) Neither venous nor arterial b) Arterial insufficiency c) Trauma d) Venous insufficiency

b) Arterial insufficiency Explanation: Characteristics of arterial insuffiency ulcers include location at the tips of the toes, extreme painfulness, and circular shape with pale to black ulcer bases. Ulcers caused by venous insufficiency will be irregular in shape, minimal pain if superficial (can be painful), and usually located around the ankles or the anterier tibial area.

Which of the following is a characteristic of an arterial ulcer? a) Brawny edema b) Border regular and well demarcated c) Ankle-brachial index (ABI) > 0.90 d) Edema may be severe

b) Border regular and well demarcated Explanation: Characteristics of an arterial ulcer include a border that is regular and demarcated. Brawny edema, ABI > 0.90, and edema that may be severe are characteristics of a venous ulcer.

The nurse is caring for a patient diagnosed with pericarditis. What serious complication should this patient be monitored for? a) Left ventricular hypertrophy b) Cardiac tamponade c) Decreased venous pressure d) Hypertension

b) Cardiac tamponade The inflammatory process of pericarditis may lead to an accumulation of fluid in the pericardial sac (pericardial effusion) and increased pressure on the heart, leading to cardiac tamponade (see Chapter 29).

A 73-year-old male client is diagnosed with dilated cardiomyopathy. The nurse is aware that which of the following is the most likely cause of his condition? a) Scleroderma b) Chronic alcohol abuse c) Previous myocardial infarction d) Heredity

b) Chronic alcohol abuse Chronic alcohol ingestion is one of the main causes of dilated cardiomyopathy. Other causes include history of viral myocarditis, an autoimmune response, and exposure to other chemicals in addition to alcohol. Heredity is considered the main cause of hypertrophic cardiomyopathy. This a connective tissue disorder is thought to cause restrictive cardiomyopathy. Scar tissue that forms after a myocardial infarction is thought to be a cause of restrictive cardiomyopathy.

Which of the following is a term used to describe the splitting or separating of fused cardiac valve leaflets? a) Chordoplasty b) Commissurotomy c) Valvuloplasty d) Annuloplasty

b) Commissurotomy Commissurotomy is the splitting or separating of fused cardiac valve leaflets. Annuloplasty is a repair of a cardiac valve's outer ring. Chordoplasty is repair of the stringy, tendinous fibers that connect the free edges of the atrioventricular valve leaflets to the papillary muscle. Valvuloplasty is a repair of a stenosed or regurgitant cardiac valve by commissurotomy, annuloplasty, leaflet repair, or chordoplasty.

The nurse practitioner inspects the patient's skin during a physical examination. She is looking for any abnormalities, especially skin findings associated with cardiovascular disease. The nurse notes a bluish tinge in the buccal mucosa and the tongue. She knows this is probably due to: a) Intermittent arteriolar vasoconstriction. b) Congenital heart disease. c) Peripheral vasoconstriction. d) Blood leaking outside the blood vessels.

b) Congenital heart disease. Cyanosis is due to serious cardiac disorders. A bluish tinge in the tongue and buccal mucosa are signs of central cyanosis caused by venous blood passing through the pulmonary circulation without being oxygenated. In the absence of pulmonary edema and cardiogenic shock, this sign is indicative of congenital heart disease. Refer to Table 12-3 in the text.

The nurse determines that a patient has a characteristic symptom of pericarditis. What symptom does the nurse recognize as significant for this diagnosis? a) Dyspnea b) Constant chest pain c) Uncontrolled restlessness d) Fatigue lasting more than 1 month

b) Constant chest pain The most characteristic symptom of pericarditis is chest pain, although pain also may be located beneath the clavicle, in the neck, or in the left trapezius (scapula) region. Pain or discomfort usually remains fairly constant, but it may worsen with deep inspiration and when lying down or turning.

A client is hospitalized for repair of an abdominal aortic aneurysm. The nurse must be alert for signs and symptoms of aneurysm rupture and thus looks for which of the following? a) Slow heart rate and high blood pressure b) Constant, intense back pain and falling blood pressure c) Constant, intense headache and falling blood pressure d) Higher than normal blood pressure and falling hematocrit

b) Constant, intense back pain and falling blood pressure Explanation: Indications of a rupturing abdominal aortic aneurysm include constant, intense back pain; falling blood pressure; and decreasing hematocrit.

A nurse is caring for a client with a central venous pressure (CVP) of 4 mm Hg. Which nursing intervention is appropriate? a) Call the physician and obtain an order for a fluid bolus. b) Continue to monitor the client as ordered. c) Call the physician and obtain an order for a diuretic. d) Rezero the equipment and take another reading.

b) Continue to monitor the client as ordered. Normal CVP ranges from 3 to 7 mm Hg. The nurse doesn't need to take any action other than to monitor the client. It isn't necessary to rezero the equipment. Calling a physician and obtaining an order for a fluid bolus would be an appropriate intervention if the client has a CVP less than 3 mm Hg. Administering a diuretic would be appropriate if the client had excess fluid, as demonstrated by a CVP greater than 7 mm Hg.

Patient with myocarditis are sensitive to which of the following medications? a) Corticosteroids b) Digoxin c) Lasix d) Penicillin

b) Digoxin The nurse must closely monitor these patients for digoxin toxicity, which is evidenced by arrhythmia,, anorexia, nausea, vomiting, headache, and malaise. If the cause of the myocarditis is haemolytic streptococci, penicillin will be given. The use of corticosteroids remains controversial.

A nurse is completing a head to toe assessment on a patient diagnosed with right-sided heart failure. To assess peripheral edema, which of the following areas should be examined? a) Legs, Toes b) Fingers, hands c) Under the sacrum d) Lips, earlobes

b) Fingers, hands When right-sided heart failure occurs, blood accumulates in the vessels and backs up in peripheral veins, and the extra fluid enters the tissues. Particular areas for examination are the dependent parts of the body, such as the feet and ankles. Other prominent areas prone to edema are the fingers, hands, and over the sacrum. Cyanosis can be detected by noting color changes in the lips and earlobes.

A white male, age 43, with a tentative diagnosis of infective endocarditis is admitted to an acute care facility. His medical history reveals diabetes mellitus, hypertension, and pernicious anemia; he underwent an appendectomy 20 years earlier and an aortic valve replacement 2 years before this admission. Which history finding is a major risk factor for infective endocarditis? a) Race b) History of aortic valve replacement c) History of diabetes mellitus d) Age

b) History of aortic valve replacement A heart valve prosthesis such as an aortic valve replacement is a major risk factor for infective endocarditis. Other risk factors include a history of heart disease (especially mitral valve prolapse), chronic debilitating disease, I.V. drug abuse, and immunosuppression. Although race, age, and a history of diabetes mellitus may predispose a person to cardiovascular disease, they aren't major risk factors for infective endocarditis.

Which of the following is a true statement regarding the role of baroreceptors? a) Increases in heart rate b) Initiates the parasympathetic response c) Initiates the sympathetic response d) Increases blood pressure

b) Initiates the parasympathetic response During elevations of blood pressure, the baroreceptors increase their rate of discharge. This initiates parasympathetic activity and inhibits sympathetic response, lowering the heart rate and blood pressure.

A home health nurse is teaching a client with peripheral arterial disease ways to improve circulation to the lower extremities. The nurse encourages which of the following in teaching? a) Elevation of the legs above the heart b) Keeping the legs in a neutral or dependent position c) Application of ace wraps from the toe to below the knees d) Use of antiembolytic stockings

b) Keeping the legs in a neutral or dependent position Explanation: Keeping the legs in a neutral or dependent position assists in delivery of arterial blood from the heart to the lower extremities. All the other choices will aid in venous return, but will hinder arterial supply to the lower extremities.

A nursing student is caring for a client with end-stage cardiomyopathy. The client's spouse asks the nurse to clarify one of the last treatment options available that the physician mentioned earlier. After checking with the primary nurse, the nursing student would most likely discuss which of the following? a) Chordoplasty b) Left ventricular assist device c) Annuloplasty d) Open commissurotomy

b) Left ventricular assist device When heart failure progresses and medical treatment is no longer effective, surgical intervention, including heart transplantation, is considered. Because of the limited number of organ donors, many clients die waiting. In some cases, a left ventricular assist device is implanted to support the failing heart until a suitable donor becomes available. The other three choices have to do with failing valves and valve repairs.

For patients diagnosed with aortic stenosis, digoxin would be ordered for which of the following clinical manifestations? a) Edema b) Left ventricular dysfunction c) Angina d) Dyspnea

b) Left ventricular dysfunction Digoxin may be used to treat left ventricular dysfunction, and diuretics may be used for dyspnea. Nitrates may be prescribed for the treatment of angina, but must be used with caution due to the risk of orthostatic hypotension and syncope.

The nurse is providing discharge instructions to a client with unstable angina. The client is ordered Nitrostat 1/150 every 5 minutes as needed for angina. Which side effect, emphasized by the nurse, is common especially with the increased dosage? a) Rash b) Orthostatic hypotension c) Dry mouth d) Nausea

b) Orthostatic hypotension A common side effect of Nitrostat, especially at higher dosages, is orthostatic hypotension. The action of the medication is to dilate the blood vessels to improve circulation to the heart. The side effect of the medication is orthostatic hypotension. A rash, nausea, and dry mouth are not common side effects.

Postpericardiotomy syndrome may occur in patients who undergo cardiac surgery. The nurse should be alert to which of the following clinical manifestations associated with this syndrome? a) Decreased white blood cell (WBC) count b) Pericardial friction rub c) Decreased erythrocyte sedimentation rate (ESR) d) Hypothermia

b) Pericardial friction rub Explanation: The syndrome is characterized by fever, pericardial pain, pleural pain, dyspnea, pericardial effusion, pericardial friction rub, and arthralgia. Leukocytosis (elevated WBCs) occurs, along with elevation of the ESR.

Which of the following is an action of the intra-aortic balloon pump (IABP)? a) Reduction of right ventricular afterload b) Reduction of left ventricular afterload c) Reduction of right ventricular preload d) Reduction of left ventricular preload

b) Reduction of left ventricular afterload The IABP decreases the workload of the heart by reducing left ventricular afterload. Additionally, it improves coronary artery blood flow by increasing coronary artery perfusion pressure. It does not reduce left or right ventricular preload.

You are evaluating the expected outcomes on a client who is recovering from a cardiac catheterization. What is an expected outcome that you would evaluate? a) The client and family understands the need for medication. b) The client and family understands the discharge instructions. c) The client and family understands the need to restrict activity for 72 hours. d) The client and family understands the client's CV diagnosis.

b) The client and family understands the discharge instructions. The client is relaxed and feels secure. The test is performed uneventfully or the client is stabilized when complications are managed successfully. The client and family have an accurate understanding of the diagnostic testing process and discharge instructions. The scenario does not indicate that the client has a CV diagnosis, a need for medication, or a need to restrict their activity for 72 hours.

A 24-year-old obese woman describes her symptoms of palpitations, chronic fatigue, and dyspnea on exertion to the cardiologist. Upon completing the examination, the cardiologist schedules a procedure to confirm his suspected diagnosis. What diagnostic would you expect him to prescribe? a) Radionuclide angiography b) Transesophageal echocardiography c) Electrocardiography d) Chest radiograph

b) Transesophageal echocardiography TEE involves passing a tube with a small transducer internally from the mouth to the esophagus to obtain images of the posterior heart and its internal structures from the esophagus, which lies behind the heart. TEE provides superior views that are not possible using standard transthoracic echocardiography. Clients whose chests are rotund or who are obese are candidates for TEE. Chest radiography and fluoroscopy determine the size and position of the heart and condition of the lungs. The radionuclide technetium-99m is used to detect areas of myocardial damage. The radionuclide thallium-201 is used to diagnose ischemic heart disease during a stress test. Electrocardiography (ECG) is the graphic recording of the electrical currents generated by the heart muscle.

The patient has had biomarkers drawn after complaining of chest pain. Which diagnostic of myocardial infarction remains elevated for as long as 3 weeks? a) Total CK b) Myoglobin c) Troponin d) CK-MB

b) Troponin Explanation: Troponin remains elevated for a long period, often as long as 3 weeks, and it therefore can be used to detect recent myocardial damage. Myoglobin returns to normal in 12 hours. Total CK returns to normal in 3 days. CK-MB returns to normal in 3 to 4 days

Myocarditis is most commonly caused by which of the following? a) Bacterial infection b) Viral infection c) Toxic agents d) Immune-mediated mechanisms

b) Viral infection Myocarditis is an inflammation of the heart muscle, commonly resulting from viral infection. It may also be caused by bacterial infections, immune-mediated mechanisms, and toxic agents.

Following a coronary artery bypass graft, a client begins having chest "fullness" and anxiety. The nurse suspects cardiac tamponade and prints a lead II electrocardiogram (ECG) strip for interpretation. In looking at the strip, the change in the QRS complex that would most support her suspicion is: a) narrowing complex. b) amplitude decrease. c) widening complex. d) amplitude increase.

b) amplitude decrease. An amplitude decrease would support the nurse's suspicion because fluid surrounding the heart, such as in cardiac tamponade, suppresses the amplitude of the QRS complexes on an ECG. Narrowing and widening complexes as well as an amplitude increase aren't expected findings on the ECG of an individual with cardiac tamponade.

Two days after undergoing a total abdominal hysterectomy, a client complains of left calf pain. Venography reveals deep vein thrombosis (DVT). When assessing this client, the nurse is likely to detect: a) loss of hair on the lower portion of the left leg. b) left calf circumference 1" (2.5 cm) larger than the right. c) pallor and coolness of the left foot. d) a decrease in the left pedal pulse.

b) left calf circumference 1" (2.5 cm) larger than the right. Explanation: Signs of DVT include inflammation and edema in the affected extremity, causing its circumference to exceed that of the opposite extremity. Pallor, coolness, decreased pulse, and hair loss in an extremity signal interrupted arterial blood flow, which doesn't occur in DVT.

The scientific rationale supporting the administration of beta-adrenergic blockers is the drugs' ability to

block sympathetic impulses to the heart

A client complains of leg pain brought on by walking several blocks — a symptom that first arose several weeks earlier. The client's history includes diabetes mellitus and a two-pack-per-day cigarette habit for the past 42 years. The physician diagnoses intermittent claudication and orders pentoxifylline (Trental), 400 mg three times daily with meals. Which instruction concerning long-term care should the nurse provide? a) "Reduce your level of exercise." b) "See the physician if complications occur." c) "Practice meticulous foot care." d) "Consider cutting down on your smoking."

c) "Practice meticulous foot care." Explanation: Intermittent claudication and other chronic peripheral vascular diseases reduce oxygenation to the feet, making them susceptible to injury and poor healing. Therefore, meticulous foot care is essential. The nurse should teach the client to bathe his feet in warm water and dry them thoroughly, cut the toenails straight across, wear well-fitting shoes, and avoid taking medications without the approval of a physician. Because nicotine is a vasoconstrictor, this client should stop smoking, not just consider cutting down. Daily walking is beneficial to clients with intermittent claudication. To evaluate the effectiveness of the therapeutic regimen, this client should see the physician regularly, not just when complications occur.

A nurse is teaching a patient about valve replacement surgery. Which statement by the patient indicates an understanding of the benefit of an autograft replacement valve? a) "The valve is from a tissue donor, and I will not need to take any blood thinning drugs with I am discharged." b) "The valve is mechanical, and it will not deteriorate or need replacing." c) "The valve is made from my own heart valve, and I will not need to take any blood thinning drugs when I am discharged." d) "The valve is made from a pig tissue, and I will not need to take any blood-thinning drugs when I am discharged."

c) "The valve is made from my own heart valve, and I will not need to take any blood thinning drugs when I am discharged." Autografts (i.e., autologous valves) are obtained by excising the patient's own pulmonic valve and a portion of the pulmonary artery for use as the aortic valve. Anticoagulation is unnecessary because the valve is the patient's own tissue and is not thrombogenic. The autograft is an alternative for children (it may grow as the child grows), women of childbearing age, young adults, patients with a history of peptic ulcer disease, and people who cannot tolerate anticoagulation. Aortic valve autografts have remained viable for more than 20 years.

In a patient with a bypass graft, the distal outflow vessel must be at least what percentage patent for the graft to remain patent? a) 40 b) 30 c) 50 d) 20

c) 50 Explanation: The distal outflow vessel must be at least 50% patent for the graft to remain patent.

The nurse is aware that statistics show an increase in the prevalence of infective endocarditis among older adults. Which of the following factors places older adults at risk for developing infective endocarditis? a) Higher rate of tuberculosis b) A greater incidence of a history of repaired congenital heart defects c) An increased use in the number of prosthetic valve replacements d) An increase in IV drug use

c) An increased use in the number of prosthetic valve replacements The prevalence of infective endocarditis among older adults has increased, due in part to the increased number of prosthetic valve replacements, including replacements for older adults, and an increase in hospital-acquired bacteremia. While history of a repaired congenital heart defect does place a client at greater risk for developing infective endocarditis in the future, it has not been shown as a contributing factor in the prevalence of infective endocarditis among older adults. IV drug use and IV drug abuse places individuals at greater risk for infective endocarditis. However, this risk has not been attributed to an increase in its prevalence among older adults. Tuberculosis is known to contribute to pericarditis among the general population and not specific to the older adult client.

The nurse is auscultating the heart sounds of a patient with mitral stenosis. The pulse rhythm is weak and irregular. What rhythm does the nurse expect to see on the electrocardiogram (ECG)? a) First-degree atrioventricular block b) Ventricular tachycardia c) Atrial fibrillation d) Sinus dysrhythmia

c) Atrial fibrillation In mitral stenosis, the pulse is weak and often irregular because of atrial fibrillation (caused by strain on the atrium).

When teaching a client with peripheral vascular disease about foot care, a nurse should include which instruction? a) Avoid using cornstarch on the feet. b) Avoid using a nail clipper to cut toenails. c) Avoid wearing canvas shoes. d) Avoid wearing cotton socks.

c) Avoid wearing canvas shoes. Explanation: The client should be instructed to avoid wearing canvas shoes. Canvas shoes cause the feet to perspire, and perspiration can cause skin irritation and breakdown. Cotton and cornstarch absorb perspiration. The client should be instructed to cut toenails straight across with nail clippers.

The nurse is caring for a client with an elevated blood pressure and no previous history of hypertension. At 0900, the blood pressure was 158/90 mm Hg. At 0930, the blood pressure is 142/82 mm Hg. The nurse is most correct when relating the fall in blood pressure to which structure? a) Sympathetic nerve fibers b) Vagus nerve c) Baroreceptors d) Chemoreceptors

c) Baroreceptors Baroreceptor sense pressure in nerve endings in the walls of the atria and major blood vessels. The baroreceptors respond accordingly to raise or lower the pressure. Chemoreceptors are sensitive to pH, CO2, and O2 in the blood. Sympathetic nerve fibers increase the heart rate. The vagus nerve slows the heart rate.

Which of the following are characteristics of arterial insufficiency? a) Aching, cramping pain b) Superficial ulcer c) Diminished or absent pulses d) Pulses are present, may be difficult to palpate

c) Diminished or absent pulses Explanation: A diminished or absent pulse is a characteristic of arterial insufficiency. Venous characteristics include superficial ulcer formation, an aching and cramping pain, and presence of pulses.

A patient who has had a recent myocardial infarction develops pericarditis and complains of level 6 (on a scale of 0-10) chest pain with deep breathing. Which of these ordered pro re nata (PRN) medications will be the most appropriate for the nurse to administer? a) Morphine sulfate 6 mg IVP every 2-4 hours b) Acetaminophen (Tylenol) 650 mg per os (po) every 4 hours c) Ibuprofen (Motrin) 800 mg po every 8 hours d) Fentanyl 2 mg intravenous pyelogram (IVP) every 2-4 hours

c) Ibuprofen (Motrin) 800 mg po every 8 hours Pain associated with pericarditis is caused by inflammation, thus nonsteroidal anti-inflammatory drugs (NSAIDs) like ibuprofen are most effective. Opioid analgesics are usually not used for the pain associated with pericarditis.

Which sign or symptom suggests that a client's abdominal aortic aneurysm is extending? a) Elevated blood pressure and rapid respirations b) Decreased pulse rate and blood pressure c) Increased abdominal and back pain d) Retrosternal back pain radiating to the left arm

c) Increased abdominal and back pain Explanation: Pain in the abdomen and back signify that the aneurysm is pressing downward on the lumbar nerve root and is causing more pain. The pulse rate would increase with aneurysm extension. Chest pain radiating down the arm would indicate myocardial infarction. Blood pressure would decrease with aneurysm extension, and the respiratory rate may not be affected.

A nurse working in a cardiac step-down unit understands that the following drugs can affect the contractility of the heart. The nurse recognizes that contractility is depressed by which of the following drugs? a) Lanoxin b) Dobutrex c) Lopressor d) Intropin

c) Lopressor Contractility is depressed by beta-adrenergic blocking medications. The other choices all enhance contractility.

After a physical examination, the provider diagnosed a patient with a grade 4 heart murmur. During assessment, the nurse expects to hear a murmur that is: a) Quiet but readily heard. b) Easily heard with no palpable thrill. c) Loud and may be associated with a thrill sound similar to (a purring cat). d) Very loud; can be heard with the stethoscope half-way off the chest.

c) Loud and may be associated with a thrill sound similar to (a purring cat). Heart murmurs are characterized by location, timing, and intensity. A grading system is used to describe the intensity or loudness of a murmur. A grade 1 is very faint and difficult to describe, whereas a grade 6 is extremely loud. Refer to Box 12-3 in the text for a description of grades 1 to 6.

The nurse is caring for a client with manifestations of dilated cardiomyopathy. When planning care, which consideration would the nurse make? a) Assist client to bathroom every 2 hours. b) Assess abdominal girth daily. c) Place bed in a high or semi-high Fowler's position as needed. d) Instruct client to avoid strenuous activity.

c) Place bed in a high or semi-high Fowler's position as needed. Dilated cardiomyopathy has clinical manifestations of dyspnea on exertion and when lying down. Depending on level of dyspnea, placing the client in an upright Fowler's position is helpful. Clients with hypertrophic cardiomyopathy have syncopal episodes and can collapse following strenuous activity. Assistance with ambulation to avoid falls is helpful. Restrictive cardiomyopathy includes manifestations of ascites and assessment of abdominal girth.

Which of the following mitral valve conditions generally produces no symptoms? a) Regurgitation b) Infection c) Prolapse d) Stenosis

c) Prolapse Mitral valve prolapse is a deformity that usually produces no symptoms and has been diagnosed more frequently in recent years, probably as a result of improved diagnostic methods. Mitral valve stenosis usually causes progressive fatigue. Mitral valve regurgitation, in its acute stage, usually presents as severe heart failure. Mitral valve infection, when acute, will produce symptoms typical of infective endocarditis.

A physician orders blood coagulation tests to evaluate a client's blood-clotting ability. The nurse knows that such tests are important in assessing clients at risk for thrombi, such as those with a history of atrial fibrillation, infective endocarditis, prosthetic heart valves, or myocardial infarction. Which test determines a client's response to oral anticoagulant drugs? a) Bleeding time b) Platelet count c) Prothrombin time (PT) d) Partial thromboplastin time (PTT)

c) Prothrombin time (PT) Explanation: PT determines a client's response to oral anticoagulant therapy. This test measures the time required for a fibrin clot to form in a citrated plasma sample following addition of calcium ions and tissue thromboplastin and compares this time with the fibrin-clotting time in a control sample. The physician should adjust anticoagulant dosages as needed, to maintain PT at 1.5 to 2.5 times the control value. Bleeding time indicates how long it takes for a small puncture wound to stop bleeding. The platelet count reflects the number of circulating platelets in venous or arterial blood. PTT determines the effectiveness of heparin therapy and helps physicians evaluate bleeding tendencies. Physicians diagnose appoximately 99% of bleeding disorders on the basis of PT and PTT values.

Part of the continued management of a patient with infective endocarditis is assessment for the presence of Janeway lesions. On inspection, the nurse recognizes these lesions by identifying which characteristic sign? a) Patterns of petechiae on the chest b) Splinter hemorrhages seen under the fingernails c) Red or purple macules found on the palms of the hands d) Erythematosus modules on the pads of the fingers

c) Red or purple macules found on the palms of the hands Janeway lesions are painless, red or purple macules found on the palms and soles.

A physician admits a client to the health care facility for treatment of an abdominal aortic aneurysm. When planning this client's care, which goal should the nurse keep in mind as she formulates interventions? a) Increasing blood pressure and monitoring fluid intake and output b) Decreasing blood pressure and increasing mobility c) Stabilizing heart rate and blood pressure and easing anxiety d) Increasing blood pressure and reducing mobility

c) Stabilizing heart rate and blood pressure and easing anxiety Explanation: For a client with an aneurysm, nursing interventions focus on preventing aneurysm rupture by stabilizing heart rate and blood pressure. Easing anxiety also is important because anxiety and increased stimulation may raise the heart rate and boost blood pressure, precipitating aneurysm rupture. The client with an abdominal aortic aneurysm is typically hypertensive, so the nurse should take measures to lower blood pressure, such as administering antihypertensive agents, as ordered, to prevent aneurysm rupture. To sustain major organ perfusion, the client should maintain a mean arterial pressure of at least 60 mm Hg. Although the nurse must assess each client's mobility individually, most clients need bed rest when initially attempting to gain stability.

Which of the following teaching interventions should the nurse include in the plan of care for a patient with valvular heart disease who is experiencing pulmonary congestion? a) Teaching patients to take nitroglycerin if shortness of breath develops b) Teaching patients to drink at least 2 L of fluid daily and monitor urine output c) Teaching patients to rest and sleep in a chair or sit in bed with head elevated d) Teaching patients to report a weight gain of 3 pounds in 1 week

c) Teaching patients to rest and sleep in a chair or sit in bed with head elevated Patients who experience symptoms of pulmonary congestion are advised to rest and sleep sitting in a chair or bed with the head elevated. In addition, the nurse educates the patient to take a daily weight and report gains of 3 pounds in 1 day or 5 pounds in 1 week to the primary provider. The nurse may assist the patient with planning activity and rest periods to achieve an acceptable lifestyle.

A patient with a recent myocardial infarction was admitted to the hospital with a new diagnosis of mitral valve regurgitation. Which of the following assessment data obtained by the nurse should be immediately communicated to the health care provider? a) The patient has a palpable thrill felt over the left anterior chest. b) The patient has 4+ peripheral edema in both legs. c) The patient has crackles audible throughout the lungs. d) The patient has a loud systolic murmur all across the precordium.

c) The patient has crackles audible throughout the lungs. Acute mitral regurgitation, resulting from a myocardial infarction, usually manifests as severe congestive heart failure. Dyspnea, fatigue and weakness are the most common symptoms. Palpitations, shortness of breath on exertion and cough from pulmonary congestion also occur. Crackles that are audible throughout the lungs indicate that the patient is experiencing severe left ventricular failure with pulmonary congestion and need immediate interventions, such as diuretics.

Which of the following is the most common site for a dissecting aneurysm? a) Cervical area b) Sacral area c) Thoracic area d) Lumbar area

c) Thoracic area Explanation: The thoracic area is the most common site for a dissecting aneurysm. About one-third of patients with thoracic aneurysms die of rupture of the aneurysm.

A nurse is caring for a dying client following myocardial infarction. The client is experiencing apnea with a falling blood pressure of 60 per palpation. Which documentation of pulse quality does the nurse anticipate? a) A pulse deficit b) Weak pulse c) Thready pulse d) Bounding pulse

c) Thready pulse The nurse is most correct to anticipate a thready (barely palpable) pulse quality. A bounding pulse indicates a strong cardiac output. A weak pulse indicates a lower pulse quality. A pulse deficit occurs when the pulses between the apex of the heart differs from the radial pulse.

The nurse is awaiting results of cardiac biomarkers for a patient with severe chest pain. The nurse would identify which cardiac biomarker as remaining elevated the longest when myocardial damage has occurred? a) CK-MB b) Brain natriuretic peptide (BNP) c) Troponin T and I d) Myoglobin

c) Troponin T and I After myocardial injury, these biomarkers rise early (within 3 to 4 hours), peak in 4 to 24 hours, and remain elevated for 1 to 3 weeks. These early and prolonged elevations may make very early diagnosis of acute myocardial infarction (MI) possible and allow for late diagnosis in patients who have delayed seeking care for several days after the onset of acute MI symptoms. CK-MB returns to normal within 3 to 4 days. Myoglobin returns to normal within 24 hours. BNP is not considered a cardiac biomarker. It is a neurohormone that responds to volume overload in the heart by acting as a diuretic and vasodilator.

The nurse understands that which of the following medications will be administered for 6 to 12 weeks following prosthetic porcine valve surgery? a) Aspirin b) Digoxin c) Warfarin d) Furosemide

c) Warfarin To reduce the risk of thrombosis in patients with porcine or bovine tissue valves, warfarin is required for 6 to 12 weeks, followed by aspirin therapy. Furosemide would not be given for 6 to 12 weeks following this type of surgery. Digoxin may be used for the treatment of arrhythmias, but not just for 6 to 12 weeks.

A client with no known history of peripheral vascular disease comes to the emergency department complaining of sudden onset of lower leg pain. Inspection and palpation reveal absent pulses; paresthesia; and a mottled, cyanotic, cold, and cadaverous left calf. While the physician determines the appropriate therapy, the nurse should: a) elevate the affected leg as high as possible. b) place a heating pad around the affected calf. c) keep the affected leg level or slightly dependent. d) shave the affected leg in anticipation of surgery.

c) keep the affected leg level or slightly dependent. Explanation: While the physician makes treatment decisions, the nurse should maintain the client on bed rest, keeping the affected leg level or slightly dependent (to aid circulation) and protecting it from pressure and other trauma. Warming the leg with a heating pad (or chilling it with an ice pack) would further compromise tissue perfusion and increase injury to the leg. Elevating the leg would worsen tissue ischemia. Shaving an ischemic leg could cause accidental trauma from cuts or nicks.

A complication after cardiac surgery that is associated with an alteration in preload is:

cardiac tamponade

What is the term for the ability of the cardiac muscle to shorten in response to an electrical impulse?

contractility

The most common heart disease for adults in the United States is:

coronary artery disease.

The pain of angina pectoris is produced primarily by:

coronary vasoconstriction

Myocardial cell damage can be reflected by high levels of cardiac enzymes. The cardiac-specific isoenzyme is:

creatine kinase (CK-MB).

The nurse is caring for a client anticipating further testing related to cardiac blood flow. Which statement, made by the client, would lead the nurse to provide additional teaching? a) "The first test I am getting is an echocardiography. I am glad that it is not painful." b) "I had an ECG already. It provided information on my heart rhythm. c) "I am able to have a nuclide study because I do not have any allergies." d) "My niece thought that I would be ordered a magnetic resonance imaging even though I have a pacemaker."

d) "My niece thought that I would be ordered a magnetic resonance imaging even though I have a pacemaker." A magnetic resonance imaging (MRI) test is prohibited on clients with various metal devices within their body. External metal objects must be removed. All other options are correct statements not needing clarification.

A patient who had a prosthetic valve replacement was taking Coumadin to reduce the risk of postoperative thrombosis. He visited the nurse practitioner at the Coumadin clinic once a week. Select the INR level that would alert the nurse to notify the health care provider. a) 3.0 b) 2.6 c) 3.4 d) 3.8

d) 3.8 Coumadin patients usually have individualized target international normalized ratios (INRs) between 2 to 3.5 to maintain adequate anticoagulation. Levels below 2 to 2.5 can result in insufficient anticoagulation and levels greater than 3.5 can result in dangerous and prolonged anticoagulation.

Which of the following medications is considered a thrombolytic? a) Heparin b) Lovenox c) Coumadin d) Alteplase

d) Alteplase Explanation: Alteplase is considered a thrombolytic, which lyses and dissolves thrombi. Thrombolytic therapy is most effective when given within the first 3 days after acute thrombosis. Heparin, Coumadin, and Lovenox do not lyse clots.

The client states, "My doctor says that because I am now taking this water pill, I need to eat more foods that contain potassium. Can you give me some ideas about what foods would be good for this?" The nurse's appropriate response is which of the following? a) Bok choy, cooked leeks, alfalfa sprouts b) Cranberries, apples, popcorn c) Asparagus, blueberries, green beans d) Apricots, dried peas and beans, dates

d) Apricots, dried peas and beans, dates Apricots, dried peas and beans, dates, and kiwi contain high amounts of potassium. The other foods listed contain minimal amounts.

The health care provider documents that the patient's pulse quality is a +1 on a scale of 0 to 4. The nurse knows that this describes a pulse that is: a) Full, easy to palpate, and cannot be obliterated. b) Diminished, but cannot be obliterated. c) Strong and bounding and may be abnormal. d) Difficult to palpate and is obliterated with pressure.

d) Difficult to palpate and is obliterated with pressure. The quality of pulses is reported using descriptors and a scale of 0 to 4. The lower the number, the weaker the pulse and the easier it is to obliterate it. A +1 pulse is weak and thready and easily obliterated with pressure.

The nurse caring for a client who is suspected of having cardiovascular disease has a stress test ordered. The client has a co-morbidity of multiple sclerosis, so the nurse knows the stress test will be drug-induced. What drug will be used to dilate the coronary arteries? a) Thallium b) Ativan c) Diazepam d) Dobutamine

d) Dobutamine Drugs such as adenosine (Adenocard), dipyridamole (Persantine), or dobutamine (Dobutrex) may be administered singularly or in combination by the IV route. The drugs dilate the coronary arteries, similar to the vasodilation that occurs when a person exercises to increase the heart muscle's blood supply. Options A, B, and C would not dilate the coronary arteries.

A patient is admitted to the hospital with possible acute pericarditis and pericardial effusion. The nurse knows to prepare the patient for which diagnostic test used to confirm the patient's diagnosis? a) Chest x-ray b) Cardiac cauterization c) CT scan d) Echocardiogram

d) Echocardiogram Echocardiograms are useful in detecting the presence of the pericardial effusions associated with pericarditis. An echocardiogram may detect inflammation, pericardial effusion, tamponade, and heart failure. It may help confirm the diagnosis.

Which of the following is an early warning symptom of acute coronary syndrome (ACS) and heart failure (HF)? a) Hypotension b) Change in level of consciousness c) Weight gain d) Fatigue

d) Fatigue Fatigue is an early warning symptom of ACS, heart failure, and valvular disease. Other signs and symptoms of cardiovascular disease are hypotension, change in level of consciousness, and weight gain.

Which of the following is the hallmark symptom for peripheral arterial disease (PAD) in the lower extremity? a) Vertigo b) Dizziness c) Acute limb ischemia d) Intermittent claudication

d) Intermittent claudication Explanation: The hallmark symptom of PAD in the lower extremity is intermittent claudication. This pain may be described as aching or cramping in a muscle that occurs with the same degree of exercise or activity and is relieved with rest. Acute limb ischemia is a sudden decrease in limb perfusion, which produces new or worsening symptoms that may threaten limb viability. Dizziness and vertigo are associated with upper extremity arterial occlusive disease.

A patient's heart rate is observed to be 140 bpm on the monitor. The nurse knows that the patient is at risk for what complication? a) A stroke b) Right-sided heart failure c) A pulmonary embolism d) Myocardial ischemia

d) Myocardial ischemia As heart rate increases, diastolic time is shortened, which may not allow adequate time for myocardial perfusion. As a result, patients are at risk for myocardial ischemia (inadequate oxygen supply) during tachycardias (heart rate greater than 100 bpm), especially patients with coronary artery disease.

The nurse is caring for clients on a busy cardiac unit. Following morning assessment, the nurse would notify the physician with which of the following symptoms? a) A noted irregular pulse rate prior to Lanoxin (digoxin) administration b) Cyanosis with a pulse oximetry level of 92% c) Dyspnea when ambulating from the bathroom d) Pulsus paradoxus on vital sign assessment

d) Pulsus paradoxus on vital sign assessment Pulsus paradoxus is a difference of 10mm Hg or more between the first Korotkoff sound noting systolic blood pressure heard during expiration and the first that is heard during inspiration. Pulsus paradoxus can signal a deteriorating condition including diminished stroke volume, compromised cardiac output, and death. This would be of high priority to notify the physician.

A female client is readmitted to the facility with a warm, tender, reddened area on her right calf. Which contributing factor should the nurse recognize as most important? a) An active daily walking program b) A history of diabetes mellitus c) History of increased aspirin use d) Recent pelvic surgery

d) Recent pelvic surgery Explanation: The client shows signs of deep vein thrombosis (DVT). The pelvic area has a rich blood supply, and thrombophlebitis of the deep veins is associated with pelvic surgery. Aspirin, an antiplatelet agent, and an active walking program help decrease the client's risk of DVT. In general, diabetes mellitus is a contributing factor associated with peripheral vascular disease.

Which of the following tests used to diagnose heart disease is least invasive? a) Cardiac catheterization b) Magnetic resonance imaging c) Coronary arteriography d) Transthoracic echocardiography

d) Transthoracic echocardiography Transthoracic echocardiography uses high-frequency sound waves that pass through the chest wall (transthoracic) and are displayed on an oscilloscope. MRI uses magnetism to identify disorders that affect many different structures in the body without performing surgery. While an MRI does not expose clients to radiation, it does require intravenous infusion to instill medication and contrast medium. Cardiac catheterization requires the insertion of a long, flexible catheter from a peripheral blood vessel in the groin, arm, or neck into one of the great vessels and then into the heart. This procedure requires the instillation of a contrast medium into each coronary artery.

When the patient diagnosed with angina pectoris complains that he is experiencing chest pain more frequently even at rest, the period of pain is longer, and it takes less stress for the pain to occur, the nurse recognizes that the patient is describing which type of angina? a) Variant b) Refractory c) Intractable d) Unstable

d) Unstable Explanation: Unstable angina is also called crescendo or preinfarction angina and indicates the need for a change in treatment. Intractable or refractory angina produces severe, incapacitating chest pain that does not respond to conventional treatment. Variant angina is described as pain at rest with reversible ST-segment elevation and is thought to be caused by coronary artery vasospasm. Intractable or refractory angina produces severe, incapacitating chest pain that does not respond to conventional treatment.

A home health nurse is seeing an elderly male client for the first time. During the physical assessment of the skin on the lower legs, the nurse notes edema, brown pigmentation in the gater area, pedal pulses, and a few irregularly shaped ulcers around the ankles. From these findings, the nurse knows that the client has a problem with peripheral circulation. Which of the following does the nurse suspect? a) Trauma b) Arterial insufficiency c) Neither venous nor arterial insufficiency d) Venous insufficiency

d) Venous insufficiency Explanation: Symptoms of venous insufficiency include present pedal pulses, edema, pigmentation in gater area, and a reddish blue color. Ulcers caused by venous insufficiency will be irregular in shape and usually located around the ankles or the anterior tibial area. Characteristics of arterial insufficiency ulcers include location at the tips of the toes, great pain, and circular shape with a pale to black ulcer base.

A client with no known history of peripheral vascular disease comes to the emergency department complaining of sudden onset of lower leg pain. Inspection and palpation reveal absent pulses; paresthesia; and a mottled, cyanotic, cold, and cadaverous left calf. While the physician determines the appropriate therapy, the nurse should: a) place a heating pad around the affected calf. b) elevate the affected leg as high as possible. c) shave the affected leg in anticipation of surgery. d) keep the affected leg level or slightly dependent.

d) keep the affected leg level or slightly dependent. Explanation: While the physician makes treatment decisions, the nurse should maintain the client on bed rest, keeping the affected leg level or slightly dependent (to aid circulation) and protecting it from pressure and other trauma. Warming the leg with a heating pad (or chilling it with an ice pack) would further compromise tissue perfusion and increase injury to the leg. Elevating the leg would worsen tissue ischemia. Shaving an ischemic leg could cause accidental trauma from cuts or nicks.

Calcium channel blockers act by:

decreasing SA node automaticity.

During an initial assessment, the nurse measures the client's apical pulse and compares it to the peripheral pulse. The difference between the two is known as pulse:

deficit

Within the heart, several structures and several layers all play a part in protecting the heart muscle and maintaining cardiac function. The inner layer of the heart is composed of a thin, smooth layer of cells, the folds of which form heart valves. What is the name of this layer of cardiac tissue?

endocardium

The most common nursing diagnosis for patients awaiting cardiac surgery is:

fear related to the surgical procedure.

A candidate for percutaneous transluminal coronary angioplasty (PTCA) is a patient with coronary artery disease who:

has at least 70% occlusion of a major coronary artery.

The nurse cares for a client in the emergency department who has a B-type natriuretic peptide (BNP) level of 115 pg/mL. The nurse recognizes that this finding is most indicative of which condition?

heart failure

Each chamber of the heart has a particular role in maintaining cellular oxygenation. Which chamber is responsible for pumping blood to all the cells and tissues of the body?

left ventricle

The most common site of myocardial infarction is the:

left ventricle.

An intravenous analgesic frequently administered to relieve chest pain associated with myocardial infarction is

morphine sulfate

The most common vasodilator used to treat myocardial pain is:

nitroglycerine.

Extremity paresthesia, dysrhythmias (peaked T waves), and mental confusion after cardiac surgery are signs of electrolyte imbalance related to the level of

potassium

The balloon on the distal tip of a pulmonary artery catheter is inflated and the pressure is measured. What is the term for the measurement obtained?

pulmonary artery wedge pressure

During auscultation of the lungs, what would a nurse note when assessing a client with left-sided heart failure?

wheezes with wet lung sounds

The nurse expects a postoperative PTCA patient to be discharged

within 24 hours of the procedure


Kaugnay na mga set ng pag-aaral

STATS EXAM, Chapter 3 (Quantitative Variables), Chapter 2 (Categorical Variables)

View Set

26208-17 Conductor Terminations & Splices, Module 26206 conductor installations, 26205-20 Pull & Junction Boxes, Midterm

View Set

Chapter 24 - Urinary system - Smart book

View Set

Live Virtual Machine Lab 7.1: Module 07 Network Architecture

View Set

Chapter 26- Caring for a Child with Cardiovascular Conditions

View Set